You are on page 1of 41

CASTLE CONNOLLY GRADUATE BOARD REVIEW SERIES

AGAI Educational Review


Manual in Gastroenterology
Fifth Edition - 2009
WEB EDITION 5.00

M. Michael Wolfe, M.D., Editor-in-Chief


Professor of Medicine, Boston University School of Medicine
Chief, Section of Gastroenterology, Boston Medical Center

Robert C. Lowe, M.D., Associate Editor-in-Chief


Associate Professor of Medicine, Boston University School of Medicine
Director of Education, Gastroenterology, Boston Medical Center
Chapter 8:
Nonviral Liver Disease
Robert C. Lowe, MD

Contents

1. Autoimmune Hepatitis

2. Alcoholic Liver Disease

3. Nonalcoholic Fatty Liver Disease (NAFLD)

4. Primary Biliary Cirrhosis

5. Granulomatous Hepatitis

6. Primary Sclerosing Cholangitis

7. Hemochromatosis

8. Alpha-1 Antitrypsin Deficiency

9. Wilson Disease

10. Vascular Disorders of the Liver

11. References

12. Questions

CHAPTER 8: NON-VIRAL LIVER DISEASE 251


1. Autoimmune Hepatitis

There is a clear genetic predisposition to AIH, with


alleles of the MHC Class II protein determining sus-
Epidemiology

Autoimmune hepatitis (AIH) is a chronic liver dis- ceptibility to AIH. These susceptibility alleles are
order characterized by hepatic inflammation and most common in persons of northern European
fibrosis, hypergammaglobulinemia, and serum descent, but AIH has been described in members of
autoantibodies. It is an uncommon disorder which all ethnic groups. As many as 40% of patients with
occurs with a frequency of roughly 50-200 cases per AIH also have other autoimmune disorders, most
million in Northern Europe and North America, notably thyroid disease and ulcerative colitis, sug-
where it accounts for 11%-23% of cases of chronic gesting that a broader immune dysregulation is pre-
hepatitis. It is estimated that there are approximately sent in these patients. Several drugs have been
200,000 patients with AIH in the United States. The demonstrated to induce an autoimmune hepatotoxic
female-to-male ratio for this disease is estimated at reaction, and a careful medication history is neces-
4:1, and the average age of onset is between 20 and sary when evaluating patients with possible AIH
40 years. A less common form of this disease, desig- (Table 1).
nated Type 2 autoimmune hepatitis, occurs primar-
ily in children, and is uncommonly seen in the
United States.
Table 1

Drugs Associated with Chronic Hepatitis


Pathogenesis and Autoimmune Markers

The etiology of autoimmune hepatitis remains


unknown, but it is speculated that “molecular
mimicry” plays a role. This concept postulates that a
Type 1

patient is exposed to an exogenous antigen that


(Positive ANA, Type 2

shares features with a self-antigen present in or on


ASMA or anti-DNA) (Positive LKM1)

hepatocytes. An aberrant immune response is then


directed at the patient’s own hepatocytes. The trig-
Clometacin Halothane

ger antigen in AIH remains unknown, but antibod-


Methyldopa Iproniazid

ies directed toward the hepatocyte asialoglycopro-


Minocycline Dihydralazine

tein receptor are present in type I AIH, while anti-


Nitrofurantoin Tienilic acid

bodies to a member of the cytochrome P450 system


Oxyphenisatin (ticrynafen)

(CYP2D6) are present in Type II disease. In both


Benzarone

forms of AIH, the damage to hepatocytes is medi-


Diclofenac

ated by cell-mediated cytotoxicity or antibody-


Fenofibrate

dependent cell-mediated mechanisms. The intense


Germander

lymphocytic infiltrate seen in liver biopsies of


Papaverine

patients with AIH is composed mainly of CD4+ T-


Pemoline

cells, with a smaller population of CD8+ cytotoxic


Propylthiouracil

T lymphocytes. As yet, the “triggering” antigen in


AIH remains obscure. Infectious agents have been
long considered to be a possible source of antigens
in autoimmune disease, but no bacterial or viral
Clinical Features

product has been definitively identified. Interest- Many patients with autoimmune hepatitis are diag-
ingly, autoimmune hepatitis has been reported to nosed in an asymptomatic phase after the discovery
occur after acute hepatitis A infection, but no spe- by a primary physician of elevated liver enzymes.
cific antigen-antibody interaction has been eluci- The disease is often insidious and without symp-
dated in these cases. toms until late in its course, and approximately 25%
of patients will have progressed to cirrhosis by the
time of presentation. In reported series, up to 85% of
patients with AIH describe fatigue, and up to 48%

252 EDUCATIONAL REVIEW MANUAL IN GASTROENTEROLOGY


complain of vague discomfort or pain in the right females (70% of cases), with an average age of
upper quadrant. Mild pruritus or anorexia is present onset before 40 years. Type II AIH, in contrast, is
in one-third of patients. On physical examination, as characterized by a positive anti-liver/ kidney micro-
many as 78% have hepatomegaly, and more than somal antibody (LKM1). It is more common in chil-
30% have concomitant splenomegaly. Spider dren, but in Europe it accounts for up to 20% of
angiomata are seen in up to one-half of patients, and adult AIH. In the United States, however, it
this physical sign may occur in AIH in the absence accounts for only 4% of adult AIH cases. Children
of cirrhosis or portal hypertension. with autoimmune polyendocrine syndrome (APS I)
have a 15% incidence of AIH, usually Type 2. Also
Although AIH is a chronic disease, it can present as included in the diagnostic criteria are tests to rule
acute hepatitis in up to 40% of patients, including out other forms of liver disease, including viral
rare cases of fulminant hepatic failure. These serologies, iron studies, and assessment of alcohol
patients present with markedly elevated transami- consumption and possible toxic exposures. A third
nases (often >1000 U/mL), hepatic synthetic dys- type of AIH has been reported, with an antibody to
function (manifested as an elevated PT) and soluble liver antigen (Anti-SLA) as a marker of dis-
encephalopathy. These patients often have underly- ease. This condition, however, is indistinguishable
ing cirrhosis, and may have signs of portal hyper- from Type I disease, and is not thought to be a sepa-
tension or other signs of advanced liver disease. It is rate clinical entity. Variant forms of AIH exist,
important to consider autoimmune disease in the which overlap with both primary biliary cirrhosis
differential of acute liver failure, especially in a (PBC) and primary sclerosing cholangitis (PSC).
young or middle-aged woman. These variants should be suspected in patients with
cholestatic liver enzymes or evidence of bile duct
Extrahepatic features of autoimmune hepatitis inflammation on biopsy. A number of patients with
occur in 10%-50% of patients, and consist primarily chronic cryptogenic hepatitis, with inflammation
of other autoimmune conditions. The most common seen on biopsy but with negative serologic workup,
manifestations include autoimmune thyroid disease will have autoimmune hepatitis that responds to
(30%-40%), synovitis, and ulcerative colitis (6%). immunosuppressive therapy, and patients with sig-
Rarely, patients may present with frank rheumatoid nificant histologic damage should be considered for
arthritis, vitiligo, lichen planus, or features of the therapy directed toward AIH.
CREST syndrome.
Once the diagnosis is made, a liver biopsy is indicated
Laboratory testing in chronic AIH demonstrates ele- to assess the grade of inflammation and the stage of
vation of the AST and ALT, with only minor eleva- fibrosis. This information is useful both for planning
tion of the ALP. Mild hyperbilirubinemia is com- possible treatment and estimating prognosis. Typical
mon, but serum bilirubin is >3 in less than half of biopsy findings include a periportal and lobular
patients. Hypergammaglobulinemia is seen in mononuclear infiltrate with abundant plasma cells
>90% of patients, with a predominant IgG fraction. (Figures 1 and 2). The degree of fibrosis is variable,
but up to 25% of patients have established cirrhosis at
time of initial biopsy. The prognosis of untreated
autoimmune hepatitis depends on the severity of the
Diagnosis

The diagnosis of autoimmune hepatitis is made on inflammatory process within the liver. Patients with
the basis of several clinical and laboratory criteria, ALT levels >10 times normal, or those with ALT >5
and a scoring system for diagnosis has been devised times normal coupled with a serum globulin level >2
(Table 2). In Type I AIH, the ANA (antinuclear anti- times normal, have a three-year mortality of 50%.
body) and the ASMA (anti-smooth muscle anti- Biopsy findings of bridging necrosis or severe lobular
body) should be measured, and an ANA titer of inflammation also portend a poor prognosis, with
>1:40 or an ASMA titer of >1:80 are characteristic over 80% of patients developing cirrhosis within 5
of this disorder. Perinuclear antineutrophil cytoplas- years. However, more mild inflammation (periportal
mic antibodies are also present in up to 90% of hepatitis) is associated with a good prognosis and
cases. Clinically, this disease is common in adult uncommonly progresses to frank cirrhosis.

CHAPTER 8: NON-VIRAL LIVER DISEASE 253


Table 2

Scoring Criteria for the Diagnosis of Autoimmune Hepatitis

Classification Factors Score

Clinical Female Gender +2

Recent hx hepatotoxic drugs:


Yes -4
No +1

Hx alcohol use:
Yes - <25 gms / day +2
>60 gms / day -2
No +0

Concurrent autoimmune disorder +2

Complete treatment response +2

Relapse +3

Biochemical

Alkaline phosphatase-AST (ALT) ratio:


>3.0 -2
<1.5 +2

Gamma-globulin or serum IgG levels


>2.0 +3
1.5-2.0 +2
1.0 - 1.4 +1

ANA, SMA or anti-LKM1:


>1:80 +3
1:80 +2
1:40 +1

Antimitochondrial Antibodies -4

Viral markers:
Seropositive -3
Seronegative +3

HLA-DR3 or HLA-DR4 +1

Other liver-related autoantibodies +2

continued on next page

254 EDUCATIONAL REVIEW MANUAL IN GASTROENTEROLOGY


Table 2 (continued)

Scoring Criteria for the Diagnosis of Autoimmune Hepatitis

Classification Factors Score

Histologic Interface hepatitis +3

Plasmacytic infiltrate +1

Rosettes +1

No characteristic features -5

Biliary changes -3

Other features (eg, fat granulomas) -3

Adapted from Al-Khalidi JA et al. Mayo Clin Proc. 2001;76:1237-1252.


Pretreatment score: >15, definite diagnosis; 10-15, probable diagnosis.
Posttreatment score: >17, definite diagnosis; 12-17, probable diagnosis.

Treatment Figure 1

The use of immunosuppressive therapy has


markedly improved the prognosis of autoimmune
Autoimmune hepatitis

hepatitis. Current guidelines published by the


AASLD recommend treatment for the following
groups of patients with AIH: those with serum
transaminase levels greater than 10x the upper limit
of normal; those with serum transaminases greater
than 5x the ULN and serum globulin levels at least
twice the ULN; and those with liver biopsies show-
ing bridging necrosis or multi-acinar necrosis.
Patients with milder histologic features or those
with inactive cirrhosis do not require treatment.

The regimens commonly employed consist of either


prednisone alone or a combination of prednisone
and azathioprine. Combination therapy is preferred,
as azathioprine serves as a steroid sparing agent,
A B

allowing prednisone to be tapered off completely or


(Left, A.) Marked chronic necroinflammatory injury with

reduced to a low level that minimizes systemic side


prominent interface hepatitis (piecemeal necrosis) and

effects. Remission induction is possible in approxi-


lymphoplasmacytic inflammatory response. Increased

mately 65% of patients at 18 months, and 80% at 3


fibrous tissue with architectural distortion suggests early

years, with remission defined as improvement in


cirrhosis. (H&E, original mag X 28) (Right, B.) High magni-

liver enzymes (AST <2x ULN) and an improvement


fication of limiting plate area showing interface hepatitis
with many plasma cells and a rare apoptotic body. (H&E

in histology to normal or minimal inflammation.


original mag X 100)

Although transaminases may normalize within 6 to

CHAPTER 8: NON-VIRAL LIVER DISEASE 255


In addition to immunosuppressive therapies,
patients should receive vitamin D (50,000 units q
Figure 2

week) and calcium supplementation (at least 1 gram


qd) to prevent osteoporosis. If bone disease is docu-
Autoimmune hepatitis

mented, then treatment with bisphosphonates or cal-


citonin is recommended. Patients who develop cir-
rhosis and end-stage liver disease from autoimmune
hepatitis are candidates for orthotopic liver trans-
plantation, with reported 5-year survival rates of
85%-90%. Autoimmune hepatitis may recur in the
graft, however, despite the immunosuppressive reg-
imen used in the post-transplant period. Recurrent
disease, which is seen in approximately 15% of
cases, is usually mild and does not adversely influ-
ence survival.

Masson trichrome preparation showing limiting plate area


with collagen fibers (blue) surrounding many hepatocytes
including rosette formation. (Original mag X 45)

12 months, the inflammatory lesion on biopsy often


lags behind the biochemical improvement. It is rec-
ommended that therapy continue for at least 6
months after the liver enzymes normalize, and a
second liver biopsy is helpful in confirming histo-
logic improvement. Therapy may be discontinued
after 2 years if a complete remission is achieved.
Unfortunately, up to 80% of patients will relapse
within 3 years of cessation of therapy and will
require retreatment and subsequent maintenance
therapy with azathioprine and/or low dose steroids.
In the 10%-15% of patients who fail to respond to
steroids and azathioprine, several other
immunomodulators may be effective, including
cyclosporin and mycophenolate mofetil; however,
these therapies are experimental, and should be
employed in the setting of clinical trials.

Immunosuppressive therapies for AIH are compli-


cated by drug-induced side effects. The effects of
chronic steroid use are well known, including
weight gain, hypertension, diabetes, cataracts, and
osteoporosis. The cosmetic effects, including buf-
falo hump, acne, and hirsutism, may also be signifi-
cant. The adverse effects of azathioprine include
pancreatitis, bone marrow suppression, and a
slightly increased risk of lymphoid malignancy with
chronic use.

256 EDUCATIONAL REVIEW MANUAL IN GASTROENTEROLOGY


2. Alcoholic Liver Disease

serum ethanol levels are relatively low. This cytoso-


lic enzyme converts ethanol to acetaldehyde, which
Epidemiology

Alcoholism remains a significant health problem is then converted to acetate via mitochondrial alde-
worldwide; in the United States, it is estimated that hyde dehydrogenase. At higher serum levels of
more than 13 million persons currently abuse alco- ethanol, the cytochrome P450 system (particularly
hol. The health effects of alcohol are widely recog- CYP2E1) participates in metabolism, also convert-
nized, and among the public, liver disease is com- ing ethanol to acetaldehyde. In chronic alcohol use,
monly believed to occur frequently in alcoholic CYP2E1 is upregulated as much as 5- to 10-fold and
patients, such that the term “cirrhosis” is usually plays a greater role in alcohol metabolism. The third
associated only with alcohol abuse in the minds of enzymatic mechanism, utilizing catalase, is of
many Americans. Interestingly, only a minority of minor significance.
alcohol abusers develop significant liver disease,
with 10%-35% of heavy drinkers developing alco- Genetic polymorphisms play a role in the clinical
holic hepatitis, and 8%-20% progressing to cirrho- response to alcohol ingestion. Isoforms of ADH exist
sis. Given the prevalence of alcoholism, however, with different rates of enzyme activity; Asian indi-
the burden of liver disease from alcohol use is large. viduals often have an ADH isoform that causes
According to CDC figures, hepatic cirrhosis ethanol to be metabolized 20% faster than Cau-
accounts for 27,000 deaths per year, making it the casians with a different ADH isoform. Variations in
12th most common cause of mortality in the United aldehyde dehydrogenase also play an important role,
States. Approximately one-half of these deaths are as up to 50% of Asians have an isoform that slowly
directly attributable to alcoholic liver disease processes acetaldehyde, leading to accumulation and
(ALD). symptoms including flushing, tachycardia, and nau-
sea. These unpleasant effects lead to decreased alco-
hol consumption among these individuals. The gen-
der difference in gastric ADH activity, mentioned
Pathogenesis

Why only a minority of drinkers develop significant above, may also play a role in the differential suscep-
liver injury is not known, but risk factors for alco- tibility to alcohol, but this is unproven.
holic liver disease include female gender, obesity,
and coinfection with hepatitis C. Genetic factors are The toxicity of ethanol to hepatocytes is multifacto-
also implicated in the pathogenensis of alcoholic rial, but oxidative injury appears to play a major role
liver disease, but there are no definitive data on the in this process. Ethanol oxidation leads to the pro-
role of specific genes in the induction of ALD. The duction of free radicals such as the superoxide anion
most important factor in the development of ALD is and the hydroxyl radical, which promote lipid per-
the quantity of alcohol consumed. It has been esti- oxidation in cellular membranes leading to tissue
mated that consumption of 80 gm of alcohol per day inflammation and fibrosis. Oxidants may also cause
for a period of 10-12 years is a threshold for the mitochondrial dysfunction, which may promote
development of ALD. The threshold is lower in steatosis by interfering with fatty acid metabolism.
women (60 gm/day) for reasons that are incom- Chronic alcohol use also depletes cellular stores of
pletely understood. It has been thought that gastric antioxidants such as glutathione, making cells more
metabolism of alcohol, which is decreased in susceptible to oxidative stress. Ethanol use also
women, is an important factor in the increased sus- leads to abnormal activation of Kupffer cells, lead-
ceptibility to ALD, but this is unproven. It should ing to cytokine production, neutrophil chemotaxis,
also be noted that these figures come from retro- and further production of reactive oxygen species.
spective studies that may be flawed, and ALD has
been demonstrated in patients with significantly
lower levels of alcohol consumption.
Clinical Features

The clinical features of alcoholic liver disease vary


Alcohol metabolism in the liver is mediated by three greatly among patients, and many remain asymp-
enzyme systems. Alcohol dehydrogenase (ADH) is tomatic until cirrhosis and decompensated liver dis-
the major pathway for ethanol metabolism when ease develop. The most important element of the

CHAPTER 8: NON-VIRAL LIVER DISEASE 257


medical history in such patients is the discovery of palmar erythema, Dupuytren’s contractures, and the
potential alcohol abuse. The CAGE questions appearance (in men) of a female escutcheon.
(Table 3) are an easy-to-use, relatively accurate
means of ascertaining alcohol abuse during the
medical interview. A patient who responds posi-
Diagnosis

tively to two or more of these questions has a high Laboratory examination in alcoholic liver disease
likelihood of alcohol abuse. Patients with advanced usually reveals an elevated AST and ALT, with a
ALD may complain of easy bruising, increasing characteristic pattern of the AST exceeding the ALT
abdominal girth, pedal edema, pruritus, and early by a factor of 2 or more. Though not a perfect dis-
signs of encephalopathy including sleep distur- criminator, the AST/ALT ratio is reasonably predic-
bances and difficulty with concentration. The more tive of ALD, especially if the ratio is 3 or greater.
obvious signs of jaundice or active gastrointestinal The reason for this AST/ALT ratio is partially
hemorrhage are easily recognized. Patients with any understood; in some patients, it appears that a defi-
of these features usually have cirrhosis, but in some ciency in pyridoxal phosphate is responsible for this
cases acute alcoholic hepatitis can manifest with phenomenon, and repletion of this vitamin equal-
severe, but reversible, signs of liver decompensa- izes the ratio. However, this is not true in all cases,
tion. Acute alcoholic hepatitis may also be accom- so other mechanisms are clearly at work. It should
panied by severe right upper quadrant pain, nausea, be noted that the transaminase levels are almost
and other signs of an acute inflammatory response, always <400 mg/dL in pure alcoholic liver disease,
including fever and chills. and higher values should spur consideration of
coexistent liver diseases, such as drug-induced hep-
atitis from acetaminophen. Other markers of
chronic alcohol use include elevation of the serum
GGT and elevation of the mean corpuscular volume
Table 3

(MCV). In acute alcoholic hepatitis, the bilirubin is


often markedly elevated, and may reach levels
CAGE Questions

greater than 20 mg/dL. Leukocytosis is also com-


mon in alcoholic hepatitis, with typical values of
12,000-15,000/mm3. Prolongation of prothrombin
1. Have you ever felt the need to cut down on your

time is seen both in acute alcoholic hepatitis and in


drinking?

advanced liver disease, as is thrombocytopenia.


2. Have other people annoyed you by criticizing

Increased iron stores may also be seen in alcoholic


your drinking?

liver disease, and patients may have elevated ferritin


3. Have you ever felt bad or guilty about your drinking?

levels and an elevated transferrin saturation.


4. Have you ever taken a drink early in the morning

Genetic testing for hereditary hemochromatosis


(an eye-opener) to steady your nerves or get rid of

may be necessary to rule out this condition.


a hangover?

Liver biopsy in alcoholic liver disease demonstrates


Adapted from Schiff, 8th edition

The physical examination may reveal signs of alco- a variety of findings (Figure 3). The most common
holic liver disease, though many of these have poor lesion is alcoholic steatosis, with macrovesicular fat
sensitivity and specificity. Signs usually associated deposition within hepatocytes. This may occur
with decompensated cirrhosis include jaundice, within days of alcohol exposure, and is reversible
asterixis, ascites, pedal edema, spider angiomata, with abstinence. Alcoholic hepatitis, by contrast, is
and ecchymoses. In alcoholic liver disease, how- characterized by neutrophilic infiltration of the hep-
ever, these signs may appear during an episode of atic lobule and hepatocyte necrosis. Alcoholic hya-
acute alcoholic hepatitis and resolve during recov- line (Mallory bodies) may also be present. The
ery, if the patient remains abstinent from alcohol. lesion is centrilobular, affecting zone 3 predomi-
Tender hepatomegaly is often noted in acute alco- nantly, although it may involve the entire lobule.
holic hepatitis, and this may resolve with absti- Pericellular and perivenular fibrosis may also be
nence. Other signs of alcoholic liver disease include present. Fibrosis may progress over time, with the

258 EDUCATIONAL REVIEW MANUAL IN GASTROENTEROLOGY


infliximab was shown to confer a survival benefit in
a small study, while in another trial, the combination
Figure 3

of infliximab and prednisone resulted in higher


patient mortality, due primarily to increased rates of
Alcoholic hepatitis

infection. More study is needed before these pow-


erful TNF inhibitors can be recommended for the
treatment of acute alcoholic hepatitis.

Corticosteroid therapy is reserved for patients with


poor prognostic indices. The most commonly used
index is the Discriminant Factor (DF), which is cal-
culated as follows:

DF = (4.6x {PT in seconds - control}) +


(serum bilirubin in mg/dL)

Values >32 are associated with >50% mortality in


hospital, and therapy with prednisone or pred-
nisolone has been recommended for such patients. It
Many hepatocytes containing easily recognizable Mal-

should be recognized that the evidence for steroid


lory bodies. Also note the polymorphonuclear leukocyte

therapy is mixed, with recommendations being


reaction.

based on meta-analysis of pooled data. A recent


development of portal-central and portal-portal ACG guideline recommends a treatment regimen of
bridges. Eventually, dense fibrous bands form, with prednisolone 40 mg po QD for four weeks, followed
regenerating nodules characteristic of established by a tapering regimen. Prednisolone is the recom-
cirrhosis. mended agent as it does not require hepatic
metabolism for activation. Treatment is contraindi-
cated in patients with active GI hemorrhage, pancre-
atitis, renal failure and active infection.
Therapy

Abstinence from alcohol is the key to therapy in


alcoholic liver disease, as it can reverse symptoms Patients with chronic alcoholic liver disease benefit
and prolong life expectancy of patients, including from abstinence and from maintenance of good
those who have already progressed to cirrhosis. The nutrition. Several agents, including colchicine,
benefit of abstinence cannot be overstated; a study PTU, polyunsaturated lecithin, metadoxine, and S-
of patients with established alcoholic cirrhosis adenosylmethionine, are being actively studied for
reported 5-year survival rates of 63% in abstinent the treatment of chronic ALD, but there are no
patients, compared with 40% in those who contin- definitive recommendations for their use outside of
ued to drink. Significant improvement of symptoms, clinical trials.
including jaundice, ascites, and encephalopathy,
may occur with cessation of drinking. Liver transplantation is an option for patients with
advanced alcoholic liver disease. Most transplant
In acute alcoholic hepatitis, patients may benefit centers require a six-month period of abstinence
from both rigorous enteral nutrition and the use of before consideration for transplant, and patients
pentoxifylline, an inhibitor of TNF synthesis that must be involved in an ongoing alcohol cessation
demonstrated a significant survival benefit in a sin- program. Survival after transplant is similar in ALD
gle randomized controlled trial. Infliximab, a more and other liver diseases, as long as abstinence is
potent inhibitor of TNF-alpha, has been studied in maintained; recidivism rates are variable, ranging
small trials with inconsistent results. Single agent from 8%-20%, and are associated with adverse out-
comes.

CHAPTER 8: NON-VIRAL LIVER DISEASE 259


3. Nonalcoholic Fatty Liver
Disease (NAFLD)

Nonalcoholic fatty liver disease (NAFLD) is a con-


dition characterized by fatty infiltration of the liver,
Table 4

with lipid-filled vacuoles within the hepatocyte


cytoplasm. NAFLD encompasses a range of condi-
Medications Associated with NAFL

tions, ranging from simple hepatic steatosis to


steatohepatitis (NASH), in which the steatosis is
accompanied by a significant inflammatory
Amiodarone

response with changes on biopsy that mimic those


Corticosteroids

seen in alcoholic liver disease. This inflammatory


Synthetic estrogens

response may lead to the induction of liver fibrosis


Tamoxifen

and, in a minority of patients, eventual progression


Calcium channel blockers

to cirrhosis. A key feature of NAFLD is the exclu-


Chloroquine

sion of alcoholic liver disease, with alcohol con-


Methotrexate

sumption of less than 40 gm/day a typical cutoff for


Perhexilene maleate

the diagnosis of NAFLD.


Tetracycline
Salicylates

Epidemiology

NAFLD was initially thought to be a disease with a


female gender predominance, as roughly 75% of initial lesion, and is a result of insulin resistance,
cases in early series were in women. These data, which leads to increased lipolysis and fatty acid
however, are from retrospective autopsy series, and delivery to the liver. The “second” hit is the induc-
recent prospective studies in patients evaluated for tion of oxidative stress with free radical formation,
abnormal liver enzymes report a nearly equal gen- lipid peroxidation, and resultant cellular damage.
der ratio. Recent epidemiologic studies have Studies of animal models of NAFLD have identified
revealed some ethnic differences in the prevalence several possible etiologic factors, including endoge-
of NAFLD, with Latino populations having an nous ethanol production, elevated hepatic
increased risk of NAFLD relative to Caucasians and TNF-alpha levels due to endotoxemia, and mito-
African-Americans. Significant risk factors for chondrial defects in fatty acid metabolism. It is
NAFLD include diabetes mellitus, obesity, and likely that the pathogenesis of NAFLD is multifac-
hypercholesterolemia, though these factors may be torial, with a combination of metabolic derange-
absent in a significant number of patients with ments determining the presence of inflammation
biopsy-proven NAFLD. These risk factors, which and progressive fibrosis.
are components of the metabolic syndrome, are
associated not only with NAFLD, but also with an
increased risk of NASH and its complications.
Clinical Features

Drug-induced hepatotoxicity may induce histologic The majority of patients with NAFLD are asymp-
changes similar to those seen in NAFLD. Among tomatic, and the disease is often discovered when an
the common medications that induce this form of elevation of serum liver enzymes is detected. A
liver injury are amiodarone, tamoxifen, nifedipine, minority of patients, however, will complain of
and diltiazem. A more complete list of drugs caus- fatigue or vague right upper quadrant discomfort.
ing steatohepatitis is found in Table 4. Physical examination is unremarkable with the
exception of hepatomegaly, which is reported in
12%-75% of cases. Splenomegaly has been reported
in about 25% of patients. Central obesity, which is
Pathogenesis

The pathogenesis of NAFLD is incompletely under- associated with insulin-resistance and the metabolic
stood, and is the subject of active research. Cur- syndrome, is often present in patients with NAFLD.
rently, a “two-hit model of NAFLD” has been Stigmata of portal hypertension are usually absent
described. It is believed that hepatic steatosis is the unless the disease has progressed to frank cirrhosis.

260 EDUCATIONAL REVIEW MANUAL IN GASTROENTEROLOGY


Diagnosis Figure 4

There is no definitive diagnostic test for NAFLD.


Patients usually manifest elevated serum transami-
Nonalcoholic steatohepatitis (NASH)

nases, with greater elevation of the ALT level com-


pared with AST. This is in contrast to alcoholic liver
disease, in which the AST:ALT ratio is usually 2:1
or greater. A minority of patients with NAFLD have
an AST/ALT pattern similar to that seen in alcoholic
liver disease; these patients tend to have more
advanced disease with established fibrosis on
biopsy. Insulin resistance can also be estimated
using the homeostasis model assessment (HOMA),
calculated by multiplying the fasting insulin level
(U/ml) by the fasting glucose level (mmol/l), and
dividing the product by 22.5. The HOMA is an
imperfect measure of insulin resistance, but may aid
in the evaluation of patients with NASH.
Mixed macrovesicular, mid-sized and microvesicular

Patients with NAFLD may display mild elevations


steatosis, a poorly formed Mallory body (arrow) and a rare
focus of necrosis with polymorphonuclear leukocyte

of alkaline phosphatase and GGT levels (to less than


response. (H&E original mag X 50)

3 times the upper limit of normal), but abnormal


bilirubin, albumin, or prothrombin time is uncom-
mon. Given the association between NAFLD and
Figure 5

both diabetes and hyperlipidemia, it is not surpris-


ing that elevated serum glucose and lipid levels are
NASH

reported in 25%-75% of patients with NAFLD.


Viral serologies and autoimmune markers are nega-
tive, but iron studies may be abnormally elevated in
a minority of patients, and there is some evidence
that these patients may have more severe hepatic
fibrosis than those with normal iron studies. This is
controversial, however, since studies conflict as to
the role of iron in the progression of NAFLD.

Radiologic imaging can identify fatty infiltration of


the liver in some cases, and ultrasound, CT, and
MRI have all been used in this capacity. Unfortu-
nately, these modalities are only moderately sensi-
Centrilobular (Zone 3) fibrosis (blue) in nonalcoholic

tive for diagnosing hepatic steatosis, and more


steatohepatitis. This finding also may occur in alcoholic

important, these imaging studies are unable to dis-


steatohepatitis. (Masson trichrome, original mag X 50)

tinguish simple steatosis from steatohepatitis, and it


is only the latter condition that is associated with biopsy is debated, as there is no effective therapy for
progressive hepatic fibrosis and possible cirrhosis. NAFLD at this time except weight loss and control
of hyperglycemia and serum lipids, which can be
Liver biopsy is currently the only definitive method recommended for patients with suspected NAFLD
of diagnosing NAFLD (Figures 4 and 5); in addition without their undergoing the risk of liver biopsy.
to confirming the presence of steatohepatitis, biopsy The counter-argument is that patients with cirrhosis
allows assessment of the level of inflammation and on biopsy (7%-16% in case series) will benefit from
fibrosis in the liver (disease staging). The utility of closer follow-up and should have an esophagogas-

CHAPTER 8: NON-VIRAL LIVER DISEASE 261


troduodenoscopy (EGD) to identify and treat NAFLD, as with other etiologies of cirrhosis, have
esophageal varices. In addition, such patients an increased risk of hepatocellular carcinoma. Liver
should be screened periodically for hepatocellular transplantation is an option for patients with decom-
carcinoma with liver ultrasound examination every pensated cirrhosis due to NAFLD, but there is evi-
6 months and serum alpha-fetoprotein determina- dence that fatty infiltration and NASH may occur in
tion 2 to 4 times per year. the transplanted liver.

Prognosis Therapy

There is controversy over the prognosis of NAFLD, Currently, therapy for NASH includes treatment of
as definitive natural history studies have not yet been diabetes and hyperlipidemia, and a program of
conducted. Recent studies, however, have suggested weight loss in obese individuals. In some cases, a
a slightly decreased surivival in patients with loss of as little as 10% of body weight can lead to an
NAFLD. Hepatic steatosis alone is not a progressive improvement in liver enzyme levels. Unfortunately,
condition, but the presence of steatohepatitis may be there are no data on whether these therapies change
associated with fibrosis and progression to cirrhosis the natural history of steatohepatitis. It is also evi-
in a subset of patients. In existing series, severe dent that rapid weight loss may induce or worsen
fibrosis is seen on initial biopsy in 15%-50% of NAFLD, so care must be taken in advising dietary
patients, and frank cirrhosis in 7%-16%. The time to therapy. Avoidance of alcohol and known hepato-
development of cirrhosis and the clinical course of toxic agents is generally recommended, as obese
NAFLD-induced cirrhosis are not yet known, but it patients develop liver disease at a higher rate after
is clear that progressive liver disease will occur in a exposure to alcohol than do lean patients.
fraction of patients with steatohepatitis. A recent
study of 103 patients followed with serial biopsies Presently, there are no proven pharmacologic
demonstrated worsening fibrosis in 37% of patients, agents for NAFLD. Ursodiol had been proposed as
with the remainder having stable or improved histol- a possible therapeutic agent based on the results of
ogy over time. Risk factors for progressive fibrosis small studies, but a recent randomized controlled
in NASH include older age, obesity, and diabetes. trial failed to show any benefit. A promising area of
The greater number of risk factors is correlated with investigation involves the use of agents that
higher degrees of fibrosis on biopsy and presumably decrease insulin resistance. Metformin has been
with a greater risk of progression to end-stage liver shown to improve serum liver enzymes in patients
disease. Recent data suggest that many patients pre- with NAFLD, and recent studies of rosigitazone
viously characterized as having cryptogenic cirrho- and pioglitazone have demonstrated improvements
sis may in fact have cirrhosis as a result of NAFLD. in serum enzymes and histologic improvement on
This is an indirect association, since the degree of liver biopsy. Randomized controlled trials are
steatosis on biopsy may decrease with increasing needed to confirm these results before these agents
amounts of fibrosis, so that a liver specimen demon- can be recommended.
strating cirrhosis with no fatty infiltration may repre-
sent a “burned out” stage of steatohepatitis. The high
frequency of NAFLD risk factors, including obesity,
diabetes, and hypercholesterolemia in patients with
undiagnosed cirrhosis make NAFLD an attractive
possible etiology. Patients with cirrhosis due to

262 EDUCATIONAL REVIEW MANUAL IN GASTROENTEROLOGY


4. Primary Biliary Cirrhosis

Primary biliary cirrhosis (PBC) is an autoimmune with this symptom complex. Up to 60% of patients
disorder affecting primarily women, characterized are asymptomatic at diagnosis, and the disease is
by progressive destruction of intrahepatic bile ducts often recognized after an elevated alkaline phos-
by an inflammatory process that leads to fibrosis and phatase level is discovered. In patients who are
eventual cirrhosis. symptomatic, fatigue is reported in 65%-78%, and
pruritus is noted in 20%-70%. The pruritus tends to
be intermittent, occurring most often at night, and
can be quite debilitating. It should be noted that the
Epidemiology

PBC is an uncommon disease, with an estimated pruritus does not correlate with severity of disease,
prevalence of 4 to 5 cases per 100,000. It is primar- and in fact may lessen over time. A minority of
ily a disease of women, with only 10% of cases patients (10%) present with jaundice, a sign of more
occurring in males. PBC is typically diagnosed in advanced disease. Physical examination reveals
middle age, with a median age at diagnosis of 50-55 hepatomegaly in greater than 50% of patients. The
years. There is a clear hereditary component to PBC presence of splenomegaly, ascites, or edema sug-
such that 4%-6% of first degree relatives of PBC gests that the disease has progressed to cirrhosis
patients develop the disorder, with a relative risk of accompanied by portal hypertension. Skin findings
PBC that is greater than 500 times that of the gen- include hyperpigmentation (25%) and xanthelasma
eral population. (10%), and excoriations from frequent scratching.

Several autoimmune diseases are associated with


PBC. Thyroid disorders (most often Hashimoto’s
Pathogenesis

The pathogenesis of PBC is only partially under- thyroiditis) are seen in 15% of patients with PBC.
stood. Liver biopsy in PBC demonstrates a dense Sjögren syndrome is seen in 50%-75% of cases,
lymphocytic infiltrate in the portal tracts, with selec- while features of CREST scleroderma are reported
tive immune destruction of biliary epithelial cells by in up to 15% of patients. Ten percent of PBC
CD4 and CD8 lymphocytes. There may also be patients develop an inflammatory arthritis. Celiac
granuloma formation from activated macrophages. disease is present in 6% of PBC patients. Interest-
The characteristic antibodies in PBC, termed anti- ingly, there appears to be an increased incidence of
mitochondrial antibodies (AMA) are directed autoimmune disorders in family members of
toward components of the pyruvate dehydrogenase patients with PBC.
complex on the inner mitochondrial membrane.
Immunologic studies have determined that the lym-
phocytes mediating the biliary damage are directed
Diagnosis

against these mitochondrial antigens as well. MHC Nearly all patients with PBC have an alkaline phos-
class 2 molecules are aberrantly expressed on bile phatase level at least 3 to 4 times normal, with simi-
duct epithelium of patients with PBC, and may lar elevations of serum GGT. Serum bilirubin levels
function to present antigen to the lymphocytes, are usually normal at diagnosis, with only 10% of
thereby contributing to the aberrant immune patients displaying frank jaundice; the presence of
response. The concept of molecular mimicry is hyperbilirubinemia is a poor prognostic sign in
invoked in PBC, as the antimitochondrial antibodies PBC. Serum transaminase levels are normal or
cross react with a number of highly conserved bac- slightly elevated. Patients with PBC usually have
terial antigens. There is also evidence that exposure elevated serum cholesterol levels, and may have an
to xenobiotics may induce the aberrant immune elevated serum globulin level, with an IgM predom-
response seen in PBC. inance. Diagnosis is confirmed by determination of
a serum anti-mitochondrial antibody (AMA) level,
which is elevated to a titer of greater than 1:40 in
90%-95% of patients with PBC. This antibody
Clinical Features

The classic description of PBC is that of a middle- reacts to antigens of the pyruvate dehydrogenase
aged woman who presents with fatigue and general- complex on the inner mitochondrial membrane. An
ized pruritus; however, only 20% of patients present elevated AMA level may also be seen in a minority

CHAPTER 8: NON-VIRAL LIVER DISEASE 263


Figure 6 Figure 8

Primary biliary cirrhosis (PBC) PBC

Chronic, nonsuppurative destructive cholangitis with an Extensive portal-portal bridging fibrosis (blue) and duc-
epithelioid granulomatous component. (H&E original topenia. (Masson trichrome. Original mag X 18)
mag X 25)

of patients with autoimmune hepatitis and rarely in


primary sclerosing cholangitis or drug-induced liver
disease. Though very sensitive for the diagnosis of
PBC, the AMA test may be negative in a variant
Figure 7

form of disease with characteristic histologic find-


ings of PBC but with negative serologic testing; this
PBC

entity is called AMA-negative PBC or, more


recently, autoimmune cholangiopathy. It should be
recognized that patients with PBC may manifest
other autoantibodies, including ANA and anti-
smooth muscle antibody (found in up to 50% of
cases of PBC). In several series, an overlap syn-
drome with features of autoimmune hepatitis and
PBC has been described, and may require therapy
aimed at both entities in order to induce remission.

Imaging studies are unrevealing in PBC, with ultra-


sound, CT scanning, and ERCP typically normal.
These studies may be useful early in the diagnostic
evaluation of PBC in order to rule out a mass lesion
A B

or large duct obstruction as a cause for an elevated


Both left (A.) and right (B.) photomicrographs display

alkaline phosphatase level.


the “florid duct” lesion of PBC (chronic nonsuppurative
destructive cholangitis) with chronic inflammatory cells

Liver biopsy in patients with PBC displays charac-


infiltrating and destroying the basement membrane and

teristic findings of bile duct destruction and an


ultimately the interlobular bile duct. (H&E left original

inflammatory infiltrate comprised of mononuclear


mag X 45; right original mag X 60)

cells. Granulomata may be present (Figure 6), but


are not required for the diagnosis. PBC may be
staged according to pathologic findings. Stage 1 (the

264 EDUCATIONAL REVIEW MANUAL IN GASTROENTEROLOGY


“florid duct lesion”) is characterized by inflamma-
tion and biliary duct destruction (Figure 7). Stage 2
Treatment

is characterized by proliferation of small bile ducts Several agents have been evaluated for the treat-
in the portal areas. The development of septal fibro- ment of PBC, but only ursodeoxycholic acid (urso-
sis characterizes stage 3, and stage 4 is frank cirrho- diol) has been proven effective in retarding the pro-
sis (Figure 8). Though useful for staging the disease, gression of this disease. At doses of
biopsy is not essential in diagnosing PBC, since a 13-15 mg/kg/day, oral ursodiol induces clinical and
significant alkaline phosphatase elevation, a positive biochemical improvement (decreased levels of
AMA, and normal imaging studies allow the diagno- alkaline phosphatase), and significantly increases
sis to be made with relative certainty. In cases where survival free of liver transplant, as demonstrated in
the diagnosis is equivocal, a biopsy is still indicated. three randomized controlled trials. Ursodiol works
Although biopsy findings correlate with the severity as both a cholerrhetic agent, increasing bile flow,
of disease, a number of predictive models exist that and as an immune modulator. It has been shown to
accurately predict the course of disease, making down-regulate the aberrantly expressed MHC class
serial biopsies unnecessary. II molecules on biliary epithelia, but its impact on
the immune dysregulation in PBC is poorly under-
stood. Other agents, including colchicines and
methotrexate, have been studied for the treatment of
Prognosis

Primary biliary cirrhosis is a slowly progressive dis- PBC, but the possible benefit of these agents is
ease that leads to hepatic cirrhosis in 10-20 years inconclusive.
after diagnosis. Currently, up to 60% of patients are
diagnosed in an asymptomatic phase, with an ele- For patients with advanced liver disease due to
vated alkaline phosphatase as the only clue to under- PBC, liver transplantation is the treatment of
lying PBC. Such patients have a good short-term choice. Prognosis after transplant is excellent, with
prognosis, with 5-year survival >90%. The duration 5-year survival rates of greater than 80%. PBC
of the asymptomatic period is very variable, but once recurs in the transplanted liver in approximately
symptoms appear, survival rates decrease relative to 20% of cases, but recurrence does not appear to
the general population. For patients with PBC, a adversely affect survival.
number of predictive models have been developed to
aid in predicting survival, with most models includ- Primary biliary cirrhosis is associated with many
ing age, bilirubin level, prothrombin time, and serum complications common to the cholestatic liver dis-
albumin, among other variables (Table 5). These eases. Osteoporosis is reported in 30%-50% of
models are important in making decisions regarding patients with PBC, and it is recommended that all
referral for consideration of liver transplantation in patients with PBC receive supplemental calcium
patients with advanced disease. and vitamin D, in doses similar to those used in
postmenopausal women. When osteoporosis is evi-
dent, bisphosphonate therapy with alendronate
appears to be beneficial. This therapy should be
used with caution in patients with cirrhosis and por-
Table 5

tal hypertension, as alendronate can induce ulcera-


tion of the esophagus and promote hemorrhage
Mayo Clinic Prognostic Model for PBC

from esophageal varices.

Pruritus is a significant problem for many patients


Risk score = 0.871 x log (serum bilirubin in

with PBC. Ursodiol therapy may reduce symptoms,


mg/dL)

but this effect has not been consistent. Agents that


-2.53 x log (albumin in g/dL)

have been used successfully to ameliorate pruritus


+0.039 x (age in years)

include antihistamines, cholestyramine, and


+2.38 x log

rifampin. Antihistamines are largely ineffective in


(prothrombin time in seconds)

PBC, while cholestyramine may ameliorate pruri-


+0.859 (if edema is present)

CHAPTER 8: NON-VIRAL LIVER DISEASE 265


5. Granulomatous Hepatitis

tus, but its use is complicated by abdominal bloat-


ing, pain, and diarrhea. Rifampin has been effective
Introduction

in reducing pruritus, but patients must be monitored The term “granulomatous hepatitis” refers to the his-
for bone marrow and liver toxicity. Opioid antago- tologic finding of epithelioid granulomas in the liver
nists, such as naltrexone and nalmephene, have also parenchyma obtained at biopsy. It is not a diagnosis
shown promise in the treatment of cholestasis- in itself, and a careful workup is required to look for
induced pruritus. the treatable causes of this pathologic finding. Gran-
ulomatous hepatitis is most often seen in patients
Hypercholesterolemia is present in 85% of patients who are evaluated for an elevated alkaline phos-
with PBC. Early in the disease, levels of HDL are phatase level, as this enzyme is most often abnormal
elevated, but with time LDL levels also rise. How- in these patients; serum transaminases, however,
ever, patients with PBC do not appear to have an may also be elevated. Often, these patients are
increased risk of atherosclerosis or coronary artery asymptomatic, but symptoms may be present,
disease. A number of patients will develop xanthe- including fever, malaise, weight loss, and right upper
lasma as a result of hyperlipidemia. The lipid disor- quadrant discomfort. Once biliary obstruction and
der in PBC is often treated with cholestyramine, liver masses are excluded by imaging studies, and an
which may also ameliorate pruritus in symptomatic AMA test rules out PBC as the etiology of the ele-
patients. Other cholesterol-lowering agents may be vated ALP, a liver biopsy should be performed.
used in patients with PBC, with the usual recom-
mended monitoring for hepatotoxicity depending The biopsy itself, though significant for granuloma-
upon the agent used. tous inflammation, is often not diagnostic. Patho-
logic features, however, may suggest a diagnosis.
Fat soluble vitamin deficiency occurs in up to 20% Caseating granulomas are often seen in mycobacte-
of patients with PBC, with deficiencies of vitamins rial infection, while noncaseating granulomas are
A, D, and K occurring most commonly. Frank steat- more often seen in sarcoidosis or drug reactions.
orrhea, however, is uncommon. Fibrin ring granulomas are characterized by a cen-
tral fibrous ring surrounding an empty vacuole, and
are seen in Q fever.

Lipogranulomas have a central lipid-filled vacuole,


and are seen in patients who ingest mineral oil. For-
eign material may also be seen within the granulo-
mata. Talc within granulomas suggests a history of
IVDU, while silica or anthracotic bodies suggest
occupational exposure to silicates or coal products.
Shistosome eggs may be seen within granulomas,
and fungal forms are occasionally noted on silver
staining. Unfortunately, the most common finding is
a nonspecific granulomatous inflammation, making
a thorough search for clinical clues mandatory.

The differential diagnosis of granulomatous liver


disease is outlined in Table 6. In the United States,
sarcoidosis, drug toxicity, and mycobacterial dis-
eases are the most common etiologies of liver gran-
ulomas. Once granulomata are identified on liver
biopsy, workup should include a careful history
with attention to occupational exposures (beryllium,
silica), animal exposures (brucellosis, Q fever) and
travel (to areas endemic for fungal or parasitic dis-

266 EDUCATIONAL REVIEW MANUAL IN GASTROENTEROLOGY


tuberculosis, but in only 25% of cases with pul-
monary involvement alone. Staining of liver granu-
Table 6

lomas for AFB is not typically diagnostic. Patients


with HIV infection and hepatic granulomas may
Causes of Granulomatous Hepatitis

have TB, atypical mycobacterial infection, viral


infection with CMV or HSV, or fungal diseases.
When infectious agents are suspected, serologic
Common (in United States)

testing, blood and tissue cultures, and direct staining


of the liver biopsy specimen may be revealing.
Sarcoidosis
Tuberculosis

Treatment is aimed at the underlying etiology of the


Primary biliary cirrhosis

liver findings. It should be noted that sarcoid liver


Drug effect

disease, though common in patients with sarcoido-


sis, is usually benign and does not cause progressive
Less Common

liver damage except in a minority of cases. Steroid


Atypical mycobacteria (in immunocompromized

therapy is not recommended unless there is ongoing


patients)

chronic hepatitis and fibrosis in addition to the gran-


Secondary syphilis

ulomatous inflammation.
Fungal infections: histoplasma, coccidioides,
cryptococcus

A subset of patients with no obvious etiology of


Miscellaneous infections: brucellosis, Q fever,

hepatic granulomata are considered to have idio-


schistosomiasis, leprosy, leshmaniasis

pathic granulomatous hepatitis. Corticosteriods and


Exposures: beryllium, copper, talc

methotrexate have been used to treat this condition,


Hodgkin lymphoma

with resolution of symptoms and an improvement in


Viral infection: CMV, EBV

liver biochemical tests and histology. It is impor-


tant, however, to completely rule out infectious eti-
eases). Physical examination is often unrevealing, ologies before embarking on an immunosuppres-
though patients may have hepatomegaly or sive regimen.
splenomegaly. A careful drug history is essential,
with attention to the drugs most often associated
with granulomatous disease, including: allopurinol;
phenytoin; carbamazepine; hydralazine; pro-
cainamide; amoxicillin-clavulinic acid; and sulfon-
amides. A chest x-ray may reveal mediastinal
adenopathy or other signs of sarcoid lung disease.
An ACE level is elevated in 50%-85% of patients
with sarcoidosis, but similar elevations are noted in
other granulomatous diseases, making this serologic
test insensitive and nonspecific. Hepatic granulo-
mas may also be present in patients with Hodgkin
disease, and may occur prior to the onset of classic
symptoms or lymphadenopathy. Thus, patients with
fevers, wasting, and adenopathy, mediastinal or oth-
erwise, need to be evaluated for possible lymphoid
malignancy.

Tuberculosis should always be considered in


patients with hepatic granulomas, especially if there
is evidence of caseation. Liver involvement is seen
in more than 90% of patients with disseminated

CHAPTER 8: NON-VIRAL LIVER DISEASE 267


6. Primary Sclerosing Cholangitis

Epidemiology Table 7

Primary sclerosing cholangitis (PSC) is an autoim-


mune condition characterized by inflammation and
Mayo Clinic PSC Risk Analysis

fibrosis of medium and large bile ducts, leading over


time to biliary cirrhosis. It is associated most closely
with inflammatory bowel disease, especially ulcera-
Risk score R = (0.06 x (age in years)) + (0.85 x

tive colitis (UC). Although only 2%-7% of patients


(LN (bilirubin value)) - (4.39 x

with UC develop PSC, more than 75% of all


(LN (hemoglobin

patients with sclerosing cholangitis have underlying


value)) + (0.51 x (biopsy value))

UC. Crohn disease is less commonly associated


+ (1.59 x (inflammatory bowel

with PSC, being seen in 10%-15% of cases.


disease value))

Tobacco use, which has been shown to be protective


against UC, also appears to decrease the incidence
R Value Risk Group

of PSC, though smoking may increase the risk of


bile duct malignancy in patients who already have
≤-5.14 Low

PSC. Primary sclerosing cholangitis is twice as


common in males as in females, and the average age
-5.13 to -3.26 Intermediate

of onset is in the mid-40s. ≥-3.25 High

Pathogenesis

The pathogenesis of PSC is unknown, but it is


believed to be an autoimmune phenomenon, given elevation of serum transaminases. Serum bilirubin
its association with ulcerative colitis and its charac- is normal until late in the course of PSC. A quarter
teristic pathologic findings of lymphocytic infiltra- of patients will have an elevated globulin with an
tion and destruction of small and large bile ducts. IgM predominance. Unlike PBC, however, antimi-
The associated hypergammaglobulinemia (of the tochondrial antibodies are rarely positive. A posi-
IgM type) and the frequent finding of autoantibod- tive antineutrophil cytoplasmic antibody (p-ANCA)
ies (p-ANCA in particular) support this concept. is found in up to 80% of patients with PSC, but this
There has been speculation that bacterial transloca- is not a specific marker as it may be present in both
tion from the inflamed bowel in UC may induce the PBC and autoimmune hepatitis. In addition, other
immune response in PSC, but there is no definitive autoimmune markers may be positive in PSC,
data to support this conjecture. A lesion similar to including ANA (35%-50%) and ASMA (10%).
PSC can be caused by cryptosporidial and CMV
infection in patients with AIDS (the so-called The course of PSC is slowly progressive, with the
“AIDS cholangiopathy”), but no infectious agent eventual development of biliary cirrhosis, portal
has been implicated in primary sclerosing cholangi- hypertension, and liver failure. Median survival has
tis. An association between PSC and polymor- been reported as 12 years in older studies, which
phisms of the TNF alpha receptor have been included many patients with advanced disease at
described, and specific MHC alleles may also be diagnosis. Currently, many patients are identified in
associated with susceptibility to PSC. an asymptomatic stage, and median survival is
reported to be approximately 17 years. Clinically,
patients are often asymptomatic at diagnosis, but
within 5-6 years the majority have overt symptoms
Clinical Features

Patients are usually asymptomatic at the time of ini- of PSC. Nonspecific symptoms of PSC include
tial diagnosis, with abnormal liver enzymes provid- fatigue and pruritus, and up to 50% may have an
ing the only clue to the presence of PSC. An ele- abnormal physical examination with hepatomegaly
vated alkaline phosphatase is the most common or splenomegaly. Patients with PSC may suffer
manifestation, often with an accompanying mild repeated episodes of jaundice or cholangitis due to

268 EDUCATIONAL REVIEW MANUAL IN GASTROENTEROLOGY


Figure 9 Figure 10

Primary sclerosing cholangitis PSC

Marked portal-periportal fibrous expansion and septal


fibrosis. Note the prominent periductal fibrosis of the large
distorted interlobular duct (upper-mid portion of photomi-
crograph) as well as an occasional small duct, chronic
inflammation and a patchy area of cholate stasis. (H&E
original mag X 16)
This ERCP image demonstrates the characteristic abnor-

Cholangiocarcinoma is an important sequela of PSC


malities of the small bile ducts, with a beaded appearance
due to multiple inflammatory strictures.

that occurs in 7%-15% of cases and is often the


bile duct obstruction from strictures. If a “dominant cause of death in these patients. It is difficult to diag-
stricture” of the CBD is seen at ERCP, dilation will nose radiologically due to the marked abnormalities
result in symptomatic relief, but the course of PSC of the biliary tree that characterize PSC. Serum
is marked by recurrent symptomatic episodes. A markers such as CA 19-9 are neither sensitive nor
prognostic model for PSC (Table 7) has been devel- specific, and ERCP with bile duct brushing and
oped and validated, and it assists clinicians in moni- biopsy have sensitivities of only 40%-70%. Thus,
toring the progression of disease and in the planning these tumors often remain undiagnosed until a late
of liver transplantation when a patient develops stage and portend a very poor prognosis. In patients
end-stage liver disease. The course of PSC is inde- with PSC who undergo liver transplantation, up to
pendent of a patient’s underlying IBD, and may 10% of the explanted livers demonstrate occult foci
occur or worsen despite effective medical therapy of of cholangiocarcinoma.
IBD. PSC may develop or progress in some patients
even after total colectomy. Diagnosis

In addition to bile duct obstruction, patients with The diagnosis of PSC is made using imaging stud-
PSC are prone to gallstone disease, with both ies. Both ERCP and MRCP can make the diagnosis
cholesterol gallstones and pigment gallstones. when the characteristic findings of multiple bile
Patients with prior ileostomy from IBD may duct strictures are visible (Figure 9). In 85%-90% of
develop peristomal varices and hemorrhage. These cases, both the intrahepatic and extrahepatic bile
are very difficult to treat locally, and TIPS has been ducts are affected, with 10% showing only intra-
used to decompress these varices and decrease the hepatic duct involvement and a small fraction with
risk of bleeding. only extrahepatic involvement. These radiologic
findings are not entirely specific, and may be seen in

CHAPTER 8: NON-VIRAL LIVER DISEASE 269


7. Hemochromatosis

patients with HIV associated cholangiopathy,


cholangiocarcinoma, congenital bile duct disorders,
Epidemiology

and after biliary tract surgery. However, the pres- Hereditary hemochromatosis (HHC) is an inherited
ence of the characteristic x-ray findings in a patient disorder of iron metabolism leading to the deposi-
with cholestatic liver enzymes and inflammatory tion of excess iron in body tissues, including the
bowel disease is sufficient for the diagnosis of PSC. skin, liver, pancreas, and heart. It is important to
Liver biopsy may show a characteristic lesion with identify patients with HHC in its early stages, as
“onion skin” fibrosis surrounding small bile ducts phlebotomy can decrease iron stores and prevent
(Figure 10). This finding, however, is seen in only end-organ damage. The HFE gene has recently been
15% of biopsies in patients with PSC, and it is rarely identified as the responsible gene for HHC. Located
used to confirm the diagnosis. on chromosome 6, the HFE gene codes for a protein
similar in structure to the HLA class I molecule.
Mutations in this gene lead to a defective protein
and disordered iron metabolism; the most common
Treatment

There is currently no proven medical therapy for mutations are denoted C282Y, H63D, and S65C;
PSC, but there are data from small studies suggest- these form the basis for the current genetic test
ing that ursodiol at high doses (20-30 mg/kg/day) available for HHC. The C282Y mutation is the most
may slow the progression of PSC. A recent random- common defect, present in 80%-85% of cases of
ized controlled trial of 219 patients receiving high- HHC. The H63D mutant confers a lesser degree of
dose UDCA failed to demonstrate a significant iron overload in the homozygous state, but the com-
improvement in symptoms, laboratory parameters, pound heterozygote (C282Y/H62D) may also mani-
or survival after five years of follow-up. There was,
however, a trend towards improved survival in the
treated group. Other large, randomized controlled
Table 8

trials are underway to further evaluate the efficacy


of UDCA in PSC.
Extrahepatic Manifestations of HHC

Other medical therapies, including glucocorticoids,


methotrexate, and penicillamine, have been studied,
but have proven ineffective in the treatment of PSC.
Organ Clinical Manifestation

Endoscopic therapy is only helpful in the 20% of


Pancreas Diabetes mellitus

PSC patients who develop dominant strictures of a


large bile duct. In such cases, dilation and stenting
Heart Cardiac dysrythmias

of narrowed ducts may effectively treat cholangitis,


Dilated cardiomyopathy

and relieve pain and jaundice. Endoscopic treat-


ment, however, does not slow the inexorable pro-
Skin “Bronze” hyperpigmentation

gression of disease.
“Gray” hyperpigmentation

Liver transplantation is the only definitive therapy


Kidneys Diabetic nephropathy

for PSC, with survival rates of 90%-95% at 1 year


and 85%-88% at 5 years. PSC returns in the trans-
Pituitary Hypothyroidism

planted liver, but appears to run a mild course in


Hypogonadism

most patients. Interestingly, patients with UC and


Loss of libido

PSC who undergo transplantation have a signifi-


cantly increased risk of colon cancer and require
Joints Crystalline-induced arthropathy

yearly screening if the colon is not resected. UDCA


therapy has been shown to reduce the risk of colon
cancer in patients with PSC and UC.

270 EDUCATIONAL REVIEW MANUAL IN GASTROENTEROLOGY


fest clinically significant iron overload. HFE muta- iron sensing in the crypt enterocytes, with a subse-
tions are common in Caucasian persons of Northern quent increase in apical DMT-1 transporters on
European descent, with an allelic frequency of these cells. When these cells reach the villus sur-
1:1020 and a frequency of homozygosity of 1:200- face, they absorb iron at a high rate, despite ade-
400. The frequency of clinical iron overload, how- quate or excess total body iron stores. The relative
ever, is significantly lower than this due to incom- contribution of duodenal versus hepatocyte HFE in
plete phenotypic penetrance. Other genetic iron the pathogenesis of hemochromatosis remains an
overload syndromes, such as African iron overload area of active research.
and juvenile hemochromatosis, do not manifest
these HFE gene mutations. When these enterocytes mature and reach the villus
apex, they absorb large amounts of iron despite a
total body iron overload. Iron then deposits in tis-
sues throughout the body, and induces tissue dam-
Pathogenesis

The role of the HFE protein is becoming clearer as age via the generation of oxygen free radicals. In the
the mechanisms of intestinal iron absorption are liver, this leads to both inflammation and fibrosis
being elucidated. Total body iron stores are approxi- resulting from the activation of hepatic stellate cells.
mately 3-5 grams, with 1-2 mg absorbed each day to
balance the 1-2 mg lost each day in senescent
intestinal cells. Iron absorption by villus enterocytes
Clinical Features

in the proximal small bowel is mediated by apical The clinical manifestations of hemochromatosis are
(DMT-1) and basal (ferroprotin) iron transporters due to iron deposition in various tissues, leading to
that take in iron and excrete it into serum bound to eventual organ dysfunction. Early studies of HCC
transferrin. The rate of iron absorption is governed included only patients with advanced disease, who
by the number of transporters on the cell surface. commonly have 1 or more symptoms. In recent
studies, however, up to 75% of patients are asymp-
Recently, the role of hepcidin in the regulation of tomatic, identified only by the presence of abnormal
iron metabolism has been elucidated. Hepcidin is a LFTs or abnormal serum iron indices. Symptoms
peptide hormone produced in the liver in response usually develop between the ages of 40 and 60 years
to the presence of iron-bound transferrin in the por- in untreated patients (Table 8). Women tend to
tal circulation. Hepcidin acts at the basal surface of develop symptoms at a later age, since menstrual
enterocytes to reduce the efflux of iron, thereby blood loss slows the accumulation of iron stores
increasing intracellular iron stores. These iron-laden during the reproductive years. Diabetes mellitus
enterocytes are sloughed off in the normal cycle of secondary to pancreatic infiltration was commonly
enterocyte loss, and their iron stores are not seen in earlier studies, as was skin hyperpigmenta-
absorbed into the circulation. There may also be a tion; these 2 symptoms led to the early name
concomitant decrease in iron absorption at the villus “bronze diabetes” for HHC. Interestingly, the
surface in response to enterocyte iron loading. The increased skin pigmentation is not due to visible
HFE gene product is expressed on the hepatocyte iron deposition in the skin, but is in fact due to
surface, and may modulate the response to circulat- increased melanin production by melanocytes,
ing iron-bound transferrin. In HFE deficient indi- induced by iron overload. Also common are arthral-
viduals, the hepatocyte fails to respond normally to gias and arthritis, often localized to the second and
iron-bound transferrin, and thus produces less hep- third MCP joints of the hands. Other symptoms
cidin. Without hepcidin to modulate iron absorp- include impotence and decreased libido (due to
tion, the villus enterocytes continue to absorb iron pituitary iron deposition), congestive heart failure
despite sufficient body iron stores, leading to total from cardiac iron overload, and signs of advanced
body iron overload. liver disease from hepatic iron deposition. In
patients with HHC and cirrhosis, the incidence of
The HFE gene product is also expressed on duode- hepatocellular carcinoma is particularly high, with a
nal crypt cells, and there is an alternate hypothesis 200-fold increased risk compared to the general
that the lack of HFE on these cells leads to deranged population. More recent studies of HHC identify

CHAPTER 8: NON-VIRAL LIVER DISEASE 271


patients on the basis of abnormal serum iron studies
or genetic testing, and the frequency of symptoms
Treatment

is, as a result, significantly lower. As an example, Phlebotomy is the mainstay of therapy for HHC, and
the frequency of hepatic cirrhosis at diagnosis was effective iron reduction prevents the end-organ com-
50%-95% in early studies, but is only reported to be plications of the disease. Each unit of blood contains
3%-13% in recent series. 250 mg of iron and decreases serum ferritin by
approximately 30 ng/mL. Thus, weekly phlebotomy
is carried on for weeks to months until the serum fer-
ritin is <50. The phlebotomy interval is then length-
Diagnosis

Currently, asymptomatic patients with an elevated ened, and most patients are effectively managed with
ferritin and a transferrin saturation (serum Fe/TIBC) maintenance phlebotomy 3-6 times per year. Patients
>50% are considered at risk for HHC and require who undergo successful phlebotomy before the
confirmatory testing. In the past, liver biopsy was development of hepatic cirrhosis have survival com-
the gold standard for diagnosis. Findings on biopsy parable to age-matched controls. Patients with cir-
included stainable iron in hepatocytes (Figure 11), rhosis, despite phlebotomy, have a 10-year survival
and a hepatic iron index (HII) of >1.9 (the HII = mg of approximately 60%, with liver failure and hepato-
Fe/g liver tissue divided by patient age). Genetic cellular carcinoma being common causes of early
testing for the common mutations (C282Y, H63D, mortality. Liver transplantation for end-stage liver
and S65C) can provide a noninvasive diagnosis of disease in HCC is effective, but survival rates are
HHC. CT and MRI scans of the liver may reveal 60% and 40%-45% at 1 and 5 years, respectively.
iron overload, but these modalites are only helpful The relatively poor results with transplant are due to
when iron stores are very elevated. Biopsy still has the fact that patients with advanced HCC have other
a role in staging disease, but is reserved for patients comorbidities (especially cardiac and endocrine dis-
with clinical features that portend a risk of signifi- orders) that shorten survival.
cant liver fibrosis. These include: 1.) age >40; 2.)
elevated AST or ALT; 3.) serum ferritin >1000. When a patient is diagnosed with HHC, it is neces-
Patients who lack these risk factors do not require sary to screen family members for the disorder, since
biopsy, and may proceed directly to therapy. early identification and iron reduction prevents com-
plications. All first-degree relatives of an affected
individual should be offered screening with serum
ferritin and iron saturation. Those persons with ele-
vated markers should undergo genetic testing and
Figure 11

begin phlebotomy. Screening for the genetic defect


is not recommended due to the incomplete pene-
Genetic hemochromatosis

trance of the HHC phenotype. Pooled data from


large studies demonstrate that 25%-58% of C282Y
homozygotes have a normal ferritin and do not have
clinically significant iron overload.

Patients with elevated iron studies may also have


secondary iron overload, either due to chronic
hemolytic anemia (thalassemia, sickle cell disease)
or multiple transfusions. This is usually clinically
apparent, but liver biopsy shows preferential iron
deposition in Kupffer cells rather than hepatocytes,
and HHI is usually <1.9. Elevated ferritin levels are
also seen in other chronic liver diseases, including
NASH, alcoholic liver disease, and hepatitis C. The
degree of hepatic iron overload in these disorders is
Grade 4+ hemosiderosis in cirrhotic liver, predominantly I

usually mild, and the role of iron in the progression


hepatocytes and biliary epithelium. When present, iron-
free foci are considered to be precancerous. (Perl’s stain.
Original mag X 18)

272 EDUCATIONAL REVIEW MANUAL IN GASTROENTEROLOGY


8. Alpha-1 Antitrypsin Deficiency

of these disorders is controversial. In patients with


genetic hemochromatosis, however, it is established
Epidemiology

that both alcohol use and HCV infection increase Alpha-1 antitrypsin (A1-AT) deficiency is an inher-
the risk of progression to cirrhosis. ited enzymatic disorder that can result in both seri-
ous lung injury and end-stage liver disease. It is
inherited in an autosomal dominant fashion, and in
Caucasians in the United States, the homozygous
form is believed to occur in 1 in 1800-2000 births.
The incidence is much lower in non-European pop-
ulations. The pulmonary manifestations occur in the
majority of patients, while the hepatic disease is a
less frequent manifestation. Nonetheless, A1-AT is
the most common childhood metabolic disease of
the liver and is responsible for a significant fraction
of pediatric liver transplants.

The enzyme A1-AT is a protease inhibitor that inac-


tivates serine proteases, including neutrophil elas-
tase produced in inflammatory reactions. It serves
an important role in protecting lung tissue from
degradation by elastase. The gene for A1-AT, on
chromosome 14, exists in several allelic forms. The
protein products of the A1-AT gene are identified by
protein electrophoresis, and are given the designa-
tion Pi and the wild type protein is designated
PiMM. Although more than 75 variant forms of A1-
AT have been identified only a small subset have
pathologic significance. In clinical A1-AT defi-
ciency, the most common variant is designated
PiZZ. This protein has less than 15% of the normal
activity, and is associated with both lung and liver
disease. The Z allele of alpha-1 antitrypsin is carried
by 1%-2% of the Caucasian United States popula-
tion, and it is estimated that 80,000-100,000 persons
in the U.S. are homozygous PiZZ. Another allele,
denoted S, is a cause of disease in patients who are
compound heterozygotes with the phenotype PiSZ;
PiSS homozygotes, however, do not appear to
develop organ damage. Other mutations are less
common, and carry a variable risk of disease (Table
9).

Pathophysiology

The pathophysiology of lung disease in A1-AT defi-


ciency is due to an inability to degrade neutrophil
elastase. The enzyme is produced in large quantities
by pulmonary neutrophils, and when unopposed by
A1-AT, it leads to progressive destruction of alveo-
lar tissue and the onset of emphysema. This is par-

CHAPTER 8: NON-VIRAL LIVER DISEASE 273


Table 9

Common ALPHA 1-ATD Mutations

ALPHA 1-AT Plasma Clinical


Mutation Prevalence Concentration Manifestations

Homozygous ZZ 4% Northern Europeans Reduced by 85% Emphysema, cirrhosis


(Glu342Lys) 3% Americans and hepatocellular
carcinoma

Homozygous S 5%-10% in persons Reduced by 40% Generally not


Mutation of European descent disease-producing
(Glu264Val)

SZ mutation <5% in persons of Reduced by 40% Clinically significant,


(Glu342Lys) European descent or more especially in smokers

Siiyama variant Rare Reduced May be clinically


(Ser53Phe) significant

Malton variant Rare Reduced May be clinically


(Phe52) significant

Null variants Rare Reduced Associated with lung


but not liver disease

ticularly true of smokers, and a family history of


COPD in younger individuals should suggest possi-
Clinical Features

ble A1-AT deficiency. The liver disease in A1-AT Clinically apparent liver disease occurs in 12%-
deficiency is not caused by the lack of enzyme activ- 15% of patients with PiZZ type A1-AT deficiency,
ity, but rather by an excess of inactive protein in the though autopsy studies demonstrate cirrhosis in
hepatocytes. A1-AT is normally produced in hepa- more than 30% of PiZZ patients; this may reflect
tocyte endoplasmic reticulum, and transported to mortality due to lung disease in patients with com-
the Golgi apparatus for final processing and even- pensated and undetected cirrhosis. The variation in
tual secretion. The Z form of A1-AT undergoes clinically apparent disease likely represents differ-
abnormal folding that prevents transport of the ences in the ability of individuals to degrade intra-
enzyme out of the ER. If the hepatocyte is unable to cellular A1-AT and prevent excessive accumulation
degrade the enzyme at a sufficient rate, there is a of protein in hepatocytes. Clinical manifestations of
buildup of protein in hepatocytes that leads to hepa- A1-AT deficiency may occur in early infancy, or
tocyte damage and liver fibrosis by an as yet may remain silent until adulthood (Table 10).
unknown mechanism. The misfolded protein is visi- Neonatal cholestasis is present in approximately
ble in hepatocytes on light microscopy as intracellu- 10% of cases, and an additional 3%-7% develop
lar globules that stain positive with periodic acid clinical disease in early childhood. In later life, A1-
Schiff (PAS) and are resistant to digestion with the AT deficiency may manifest as a chronic elevation
enzyme diastase. of AST and ALT in asymptomatic patients, and it

274 EDUCATIONAL REVIEW MANUAL IN GASTROENTEROLOGY


Table 10

Clinical Manifestations of Alpha 1-ATD

Age at Presenation Clinical Presenation Other Clinical Features

Neonate 8%-12% present with prolonged cholestatic Failure to thrive, irritability,


jaundice (direct reacting bilirubin) acholic stools, bleeding
Up to 50% have elevated AST/ALT from the umbilical,
by age 3 months (often asymptomatic). stump, easy bruising,
Fulminant hepatic failure (rare) hepatomegaly, ascites, pruritus,
hypercholesterolemia

Child/Adolescent 12%-15% develop clinically evident liver


disease with hepatosplenomegaly, ascites
and variceal bleeding

Adult Up to 37% develop cirrhosis from autopsy Emphysema, bronchiectasis,


studies and pulmonary vasculitis
Hepatocellular carcinoma risk is increased may be present
(especially after age 50); may be a presenting (especially among smokers)
feature
May present as unexplained liver enzyme eleva-
tions or complications of portal hypertension

should be considered in patients who are referred the cytoplasm of hepatocytes (Figure 12). These
for evaluation of abnormal liver enzymes. Other granules are not pathognomonic of A1-AT defi-
patients present with clinical signs of cirrhosis, ciency, but their presence should lead to phenotype
including ascites, variceal hemorrhage, or jaundice. analysis of affected patients.
There is also a significantly increased risk of hepa-
tocellular carcinoma in patients with A1-AT defi-
ciency.
Treatment

Patients with A1-AT deficiency who have pro-


gressed to end-stage liver disease will benefit from
orthotopic liver transplantation, which corrects the
Diagnosis

The diagnosis of A1-AT deficiency is made by mea- metabolic defect so that disease will not occur in the
surement of a serum A1-AT level and elec- transplanted organ. Outcomes in children after
trophoretic analysis to identify the mutant alleles of transplant are good, with 1- and 5-year survivals of
A1-AT. Decreased serum levels of A1-AT are usu- 90% and 80%, respectively. There is no other avail-
ally seen in A1-AT deficiency, but normal levels do able therapy for the liver disease in A1-AT, though
not definitively exclude the diagnosis, as A1-AT is there is active research into possible gene therapy
an acute phase reactant and may reach normal levels for this condition. The lung disease in this disorder
during acute illnesses and inflammatory states. can be treated with replacement A1-antitrypsin,
Liver biopsy demonstrates the accumulation of A1- which has been shown to ameliorate the pulmonary
AT as PAS-positive, diastase resistant granules in disease and improve survival in these patients.

CHAPTER 8: NON-VIRAL LIVER DISEASE 275


9. Wilson Disease

Figure 12 Epidemiology

Wilson disease (WD) is a rare genetic defect of hep-


atic copper metabolism characterized by copper
Liver biopsy in alpha-1 antitrypsin

deposition in tissues including the liver, cornea, kid-


deficiency

ney, and central nervous system. It is inherited in an


autosomal recessive fashion with the homozygous
state occurring in approximately 1 in 30,000-50,000
The hepatocytes demonstrate the PAS (+) granules

live births. The liver manifestations of WD range


characteristic of this condition

from asymptomatic chronic hepatitis to cirrhosis


12A: low power 12B: high power

and end-stage liver disease. WD may also cause ful-


minant hepatic failure requiring emergency trans-
plantation. It is estimated that 3%-6% of patients
with fulminant hepatic failure requiring transplanta-
tion have WD as the underlying etiology. Wilson
disease should be considered in all patients with
chronically elevated serum transaminases, espe-
cially those with associated symptoms.

Pathogenesis

The Wilson disease gene (designated ATP7B) on


chromosome 13 codes for a P-type ATPase trans-
port protein in the hepatocyte. This protein trans-
ports copper into the bile and facilitates binding to
ceruloplasmin, the serum transporter of copper.
A

Typically, 1-5 mg of copper is absorbed from


dietary sources each day, and 1-2 mg is excreted
daily in the bile. In Wilson disease, copper transport
across the canalicular membrane is impaired, as is
incorporation of copper into ceruloplasmin. As a
result, copper accumulates in hepatocytes, inducing
inflammation and fibrosis, likely via the elaboration
of oxygen free radicals and lipid peroxidation.
Excess copper leaks into the bloodstream and
deposits in other tissues, including the basal ganglia
of the CNS, the cornea, kidney, and heart. More than
60 mutations of ATP7B have been identified, with
varying degrees of activity. This accounts for the
variable presentation of WD, and the large number
of mutations make genetic screening of patients and
family members difficult.

Clinical Features

Patients with Wilson disease are most often between


B

the ages of 8 and 18, but both younger and older


patients have been identified; current guidelines rec-
ommend that WD be considered in patients between
the ages of 3 and 55 with liver abnormalities of

276 EDUCATIONAL REVIEW MANUAL IN GASTROENTEROLOGY


although Wilson disease is more common in males
overall, the incidence of FHF is higher in females
Figure 13

Wilson disease with established cirrhosis with WD.

Diagnosis

WD may manifest as a neuropsychiatric illness with


associated liver abnormalities or as a purely hepatic
disorder. Chronic liver disease in WD may manifest
as abnormal liver enzymes in an asymptomatic
patient. Other patients present with established cir-
rhosis and come to attention because of jaundice,
poor synthetic function, or signs of portal hyperten-
sion. Neurologic disease may be absent in these
patients, and most patients with only hepatic WD
lack the characteristic KF rings on corneal examina-
tion. Neuropsychiatric symptoms, when present,
typically manifest in adolescence or early adult-
hood. The neurologic symptoms of WD include a
progressive movement disorder characterized by
Accumulation of copper (brick-red granules) predomi-
nantly along septa of cirrhotic nodules. (Rhodamine. Origi-

tremor, ataxia, rigidity, and dysarthria/dysphagia.


nal mag X 25)

unknown etiology. Patients may present with liver The inexorable progression of symptoms can leave
disease, neuropsychiatric disorders, or both, but the patients unable to care for themselves, despite intact
clinical features are quite variable. cognitive functioning. Up to one-third of patients
present with psychiatric symptoms, including
Most patients have some degree of hepatic disease, depression and psychosis. They are often misdiag-
and liver disease without neurologic symptoms is nosed as having schizophrenia or other primary psy-
seen in 20%-45% of cases. Isolated hepatic disease chiatric disorders, and liver enzymes should be
is more commonly seen in children, while neuro- obtained in young patients with new onset psychi-
logic and psychiatric symptoms occur more fre- atric symptoms to screen for WD. In patients with
quently in adolescents and young adults. The onset the typical neurologic syndrome, a diagnosis of WD
of symptoms is rare after age 40. Fulminant WD can be made if KF rings are present (seen in up to
presents in a manner similar to other forms of acute 90% of these patients), and the serum ceruloplasmin
hepatic failure, but several clues may be helpful in level is low. Brain MRI may show evidence of cop-
diagnosing WD in this group. Typically, serum AST per deposition in the basal ganglia, but this is not
and ALT values are less than 10 times the upper necessary for diagnosis.
limit of normal, while serum bilirubin values may
be markedly elevated. Serum alkaline phosphatase Patients with a purely hepatic presentation of WD,
is normal or in many cases significantly decreased. however, often lack KF rings (<50%), and lack
The presence of a Coombs negative hemolytic ane- symptoms specific for WD. The diagnosis of WD is
mia in a patient with fulminant hepatic failure suggested in these patients by a low serum cerulo-
(FHF) may be a sign of WD, as free copper released plasmin level, as ceruloplasmin degrades rapidly
into the bloodstream may induce severe acute when not bound to copper. However, since cerulo-
hemolysis. Serum ceruloplasmin and other serum plasmin is an acute phase reactant, it may reach low-
copper indices (discussed below) may not be diag- normal levels in up to 45% of patients with WD,
nostically helpful in fulminant WD, and Kaiser- especially in acute inflammatory states. A 24-hour
Fleischer rings may be absent, so a strong clinical urine copper determination is a more accurate diag-
suspicion of Wilson disease is advised for any nostic tool and should be considered when the clini-
young patient with these features. Interestingly, cal suspicion of WD is high, even if the serum ceru-
loplasmin is not decreased. A decreased ceruloplas-

CHAPTER 8: NON-VIRAL LIVER DISEASE 277


min is also not specific for WD, and may be seen in Trientine is another chelating agent effective in the
malnutrition, autoimmune liver disease, and in other treatment of WD. It is equal in effectiveness to peni-
genetic disorders of copper metabolism. Serum cop- cillamine with a significantly lower incidence of
per levels are also not sufficiently sensitive or spe- side effects. It may be used as a first-line therapy, or
cific to diagnose WD. Urinary copper levels in cases of intolerance to penicillamine. Oral zinc
(>100 mg in a 24-hour urine collection) have a bet- therapy (150 mg po TID) has also been shown to be
ter diagnostic accuracy, and are recommended when effective in the treatment of WD. Zinc interferes
the index of suspicion of WD is high. Liver biopsy with intestinal absorption of copper both by com-
may show nonspecific features of chronic inflam- peting for transporters in the enterocyte membrane
mation, steatosis, fibrosis, or established cirrhosis and by inducing metallothionein production. This
(Figure 13). An hepatic copper content of enzyme binds metals such as copper and prevents
>250 mcg/gram dry weight is usually seen, but there their systemic absorption. The copper adducts are
is great variability in copper deposition in WD and a later excreted via shedding of enterocytes into the
single biopsy may not be diagnostic. Thus, a combi- intestinal lumen. Zinc therapy appears to be effec-
nation of clinical, laboratory, and histologic features tive in maintaining total body copper at nontoxic
are needed to make a diagnosis of Wilson disease. levels, and it is gaining wider use in patients with
Recent guidelines recommend the use of serum WD after initial chelation therapy reduced copper to
ceruloplasmin, 24-hour urinary copper excretion, nontoxic levels. Antioxidants such as alpha-toco-
and a slit lamp exam for KF rings in any patient with pherol have been advocated as an adjunctive ther-
suspected Wilson disease. Patients with both KF apy to limit oxidative damage in WD. To date, how-
rings and a decreased ceruloplasmin are given a ever, there have been no controlled studies of
definitive diagnosis of WD; patients with only a sin- antioxidants in patients with WD.
gle suggestive test should have a liver biopsy with
quantitative copper determination. Patients being treated for WD require twice yearly
liver chemistries, CBC, ceruloplasmin, and serum
copper. A 24-hour urine copper determination
should be performed yearly, with the goal urinary
Treatment

It is important to confirm the diagnosis of WD, since copper of 200-500 mcg/day in patients on chelation
therapy can prevent the development of end-stage therapy or <75 mcg/day in patients on zinc therapy.
liver disease and may ameliorate or reverse the neu-
rologic sequelae. Avoidance of foods rich in copper The prognosis of untreated WD is poor, with an
is recommended, including shellfish, nuts, choco- inexorable progression to end-stage liver disease,
late, mushrooms, and liver. Dietary therapy, how- disabling neurologic symptoms, or both. Medical
ever, is insufficient to prevent copper overload, and therapy, if initiated prior to severe liver or neuro-
medical therapy is required. Penicillamine is an oral logic damage, confers a substantial mortality
agent that chelates serum copper and increases uri- benefit, with some studies demonstrating a cumula-
nary copper excretion. At doses of 1-2 g/day, it is tive survival in WD patients that equaled that of
very effective in decreasing total body copper stores controls. Fulminant WD, however, does not respond
in WD to nontoxic level, and chronic therapy pre- to medical therapy, and orthotopic liver transplanta-
vents the re-accumulation of toxic levels. Neu- tion is lifesaving in such cases. Liver transplantation
ropsychiatric symptoms may initially worsen, but for decompensated cirrhosis in WD is also indi-
tend to improve steadily with continued therapy. cated, with post-transplant survival reported at 80%
Unfortunately, up to 20% of patients on chronic at 1 year. Liver transplantation corrects the bio-
penicillamine therapy develop significant side chemical defect in WD, so that there is no recur-
effects, including leukopenia, thrombocytopenia, rence of disease post-transplant. Neurologic symp-
systemic lupus erythematosus, oral ulcers, and a toms may improve post-transplant as well, but this
number of other immunologic and dermatologic is not a universal occurrence.
conditions. Severe side effects necessitate cessation
of therapy.

278 EDUCATIONAL REVIEW MANUAL IN GASTROENTEROLOGY


10. Vascular Disorders of the Liver

is most marked in zone 3 of the hepatic acinus, but


can spread to encompass the entire liver and lead to
Budd-Chiari Syndrome

The Budd-Chiari syndrome (BCS) describes a frank cirrhosis.


diverse set of clinical entities that have in common
an obstruction of hepatic venous outflow. This may
occur at the level of the inferior vena cava (IVC),
Clinical Features

hepatic veins, or hepatic venules, and may be due Budd-Chiari syndrome typically presents in the
either to thrombosis or to membranous webs in the third or fourth decade, and affects women more fre-
large vessels. The clinical manifestations of Budd- quently than men. The clinical features of BCS are
Chiari syndrome depend not only on the location of variable, and range from asymptomatic abnormali-
the obstructing lesion, but also the acuity of obstruc- ties of liver enzymes to fulminant hepatic failure. In
tion. Thus, BCS is a protean syndrome that requires acute hepatic venous obstruction (20% of cases),
a high degree of suspicion if it is to be diagnosed patients usually present with severe right upper
and treated in a timely fashion. quadrant pain, hepatomegaly, nausea and vomiting,
and ascites. Jaundice may be present, but is not a
In the United States and other Western nations, BCS universal finding. Liver enzymes in acute BCS may
is largely due to thrombotic obstruction of the hep- be moderately or markedly elevated, with serum
atic veins. Up to 25% of patients with BCS have an transaminases greater than 1000 IU/mL in some
overt myeloproliferative disorder, most commonly cases. A small number of patients develop fulminant
polycythemia vera. Many patients with presumed hepatic failure in the setting of acute BCS, with
idiopathic BCS may have a subclinical hematologi- coagulopathy, encephalopathy, and jaundice. These
cal disorder. Second in frequency are the hyperco- patients typically do not survive unless they receive
agulable states, with Factor V Leiden identified as a an orthotopic liver transplant.
common cause of BCS. Other hypercoagulable
states, including Protein C and S deficiency, Budd-Chiari syndrome more commonly presents as
antithrombin III deficiency, paroxysmal nocturnal a subacute disease, with weeks of vague right upper
hemoglobinuria, and antiphospholipid antibody quadrant discomfort, hepatomegaly, and the grad-
syndrome, are also known to cause BCS. Oral con- ual onset of ascites and splenomegaly. The chronic
traceptive use, pregnancy, and the early post-partum form of BCS is insidious and remains asymp-
period have also been implicated in up to 20% of tomatic until the development of cirrhosis and por-
cases of BCS. Up to 10% of cases of BCS are due to tal hypertension, when patients present with
malignancy, especially hepatocellular and renal cell variceal bleeding, ascites, coagulopathy, or
cancer, though hepatic vein thrombosis has been encephalopathy. In chronic BCS, transaminases are
reported in patients with a wide variety of tumors. usually mildly elevated, but no specific pattern of
liver enzymes is seen.
In India, Africa, and the Far East, however, throm-
botic Budd-Chiari syndrome is uncommon, and
membranous webs in the IVC are the most common
Diagnosis

cause of hepatic outflow obstruction. The etiology The diagnosis of BCS is made using hepatic imag-
of these webs is unclear, and they may be congenital ing studies. Doppler ultrasound of the hepatic veins
or acquired. is 85%-95% sensitive for the diagnosis of BCS, but
its accuracy depends on operator experience. Mag-
In prolonged hepatic venous obstruction, regardless netic resonance venography and CT angiography
of the etiologic lesion, characteristic pathological are also useful in making a diagnosis of BCS. The
changes occur in the liver. The caudate lobe, which gold standard remains angiography, and although
drains into the IVC directly, often enlarges as blood the classic “spider web” patterns of collaterals is not
is preferentially shunted to this lobe, while the other seen in all cases, this modality is still able to diag-
portions of the liver atrophy and shrink. The histo- nose BCS with a high degree of accuracy.
logic hallmark of BCS is centrilobular congestion,
with hepatocyte necrosis and eventual fibrosis. This

CHAPTER 8: NON-VIRAL LIVER DISEASE 279


Liver biopsy is also helpful in evaluating patients
with BCS. It may confirm the diagnosis by demon-
Hepatic Veno-occlusive Disease

strating centrilobular congestion, necrosis, and Veno-occlusive disease of the liver (VOD), also
fibrosis, and, more importantly, biopsy identifies known as sinusoidal obstruction syndrome, is a vas-
patients whose BCS has progressed to cirrhosis, cular lesion characterized by obstruction of small
information which has important implications for hepatic venules by concentric narrowing and con-
therapy. nective tissue deposition. Unlike Budd-Chiari syn-
drome, venous thrombosis is not a major component
of VOD, but the clinical sequelae are similar, with
portal hypertension and varying degrees of hepato-
Treatment

The therapy of Budd-Chiari syndrome is aimed at cyte necrosis. It is most often due to a toxic injury to
relieving the vascular obstruction and preserving the hepatic vascular endothelium, with radiation
liver function by reducing hepatic venous conges- and chemotherapy the major etiologic agents. Most
tion. Medical therapy consists of anticoagulation, cases of VOD are associated with bone marrow
which is indicated in acute BCS to inhibit further transplantation, due to the combination of high dose
thrombosis and promote the lysis of existing clot. chemotherapy (usually with alkylating agents) and
Thrombolytic therapy has been used in a limited total body irradiation. Patients with pre-existing
number of cases in the acute setting, and may relieve liver disease, and those undergoing allogeneic bone
hepatic obstruction if used early in the course of marrow transplantation are at the highest risk for
BCS. Interventional radiology can play an important VOD; women appear to have a higher incidence of
role in the management of BCS. Hepatic vein angio- VOD than men. Rarely, VOD is seen after ingestion
plasty has been used successfully in BCS, and of pyrrolizidine alkaloids, which may be a compo-
although patency rates are only 50% at 2 years with nent of herbal teas (Jamaican bush tea) or other
conventional balloon angioplasty, the more recent remedies.
use of intravascular stents has demonstrated greater
patency rates of 80% at 3 years. Experience with this Clinical features of VOD are similar to those of the
technique remains limited, but the results of small Budd-Chiari syndrome. In transplant patients, VOD
case series are promising. Transjugular intrahepatic typically occurs within 3 weeks of transplantation,
portosystemic shunt (TIPS) has also been used suc- and in its acute form is characterized by ascites,
cessfully in Budd-Chiari syndrome, and clinical weight gain, abdominal pain, and hepatomegaly.
experience with this modality is growing rapidly, Serum bilirubin is usually elevated, and the risk of
such that TIPS and IVC stenting are reasonable liver failure and death is 30%-50%. More indolent
options in selected patients with BCS. forms of VOD are seen with herbal ingestion or
chronic chemotherapeutic toxicity, and may present
Surgical vascular shunts have been the standard as abnormal liver enzymes, portal hypertension, or
treatment for acute or subacute BCS. Side-to-side established cirrhosis.
portacaval shunts are most commonly performed,
though mesocaval and mesoatrial shunts are also As the pathologic process in veno-occlusive disease
used when the anatomy is less favorable. Surgical is in the terminal hepatic venules, the diagnosis of
shunting preserves hepatic function and long-term VOD is made at liver biopsy rather than at angiogra-
survival rates of 85% in selected patients have been phy. If a biopsy is considered to be too risky, clinical
reported. In patients with fulminant BCS or chronic criteria may allow a presumptive diagnosis to be
BCS that has progressed to hepatic cirrhosis and made, especially in the bone marrow transplant
end-stage liver disease, orthotopic liver transplan- patient. There is no established treatment for VOD,
tation is the treatment of choice, with 5-year sur- though the antithrombotic drug defibrotide has been
vival reported at 71%. In addition, liver transplan- used in uncontrolled trials and was associated with a
tation may cure certain hypercoagulable states, 35%-55% rate of recovery in moderate-to-severe
including Protein C or S deficiency and antithrom-
bin III deficiency.

280 EDUCATIONAL REVIEW MANUAL IN GASTROENTEROLOGY


11. References

VOD. Trials aimed at the prevention of VOD in the 1. Ericksson S. Alpha 1-antitrypsin deficiency.
transplant setting have not been definitive, though J Hepatol. 1999;30:S34-S39.
there are some data that heparin infusion or low
molecular weight heparin may reduce the incidence 2. Carrell RW, Lomas DA. Alpha 1- antitrypsin
of post-transplant VOD. Ursodiol was demonstrated deficiency—A model for conformational dis-
to decrease the incidence of VOD in bone marrow eases. N Engl J Med. 2002;346:45-53.
transplant in two studies, but not in a third large ran-
domized controlled trial. Altering the myeloablative 3. Michieletti P, Wanless TR, Katz A, et al.
chemotherapy regimens and providing liver shield- Antimitochondrial antibody negative primary
ing during irradiation may also decrease the inci- biliary cirrhosis: A distinct syndrome of
dence of VOD after bone marrow transplant. autoimmune cholangitis. Gut. 1994;35:260.

4. Czaja A, McFarlane I, Hoofnagle JH, et al.


Autoimmune hepatitis: The investigational
and clinical challenges. Hepatology.
2000;31:1194.

5. Johnson P, McFarlane IG, Williams R. Aza-


thioprine for long-term maintenance of remis-
sion in autoimmune hepatitis. N Engl J Med.
1995;333:958.

6. Tilanus H. Budd-Chiari syndrome. Br J Surg.


1995;82:1023-1030.

7. Janssen HL, GarciaPagan JC, Elias E, et al.


Budd-Chiari syndrome: A review by an expert
panel. J Hepatol. 2003;38:364-371.

8. Denninger MH, Chiat Y, Casadevall N, et al.


Cause of portal or hepatic venous thrombosis
in adults: the role of multiple concurrent fac-
tors. Hepatology. 2000;31:587-591.

9. Chalasia N, Baluyut A, Ismail A, et al.


Cholangiocarcinoma in patients with primary
sclerosing cholangitis: a multicenter case-
control study. Hepatology. 2000;31:7-11.

10. Perlmutter D. Clinical manifestations of alpha


1-antitrypsin deficiency. Gastroenterol Clin
N Am. 1995;24:27-43.

11. Poupon RE, Lindor KD, Cauch-Dudek K, et


al. Combined analysis of randomized con-
trolled trials of ursodeoxycholic acid in pri-
mary biliary cirrhosis. Gastroenterology.
1997;113:884-890.

CHAPTER 8: NON-VIRAL LIVER DISEASE 281


12. Levy C. Current management of primary bil- 23. Alvarez F, Berg PA, Bianchi F, et al. Interna-
iary cirrhosis and primary sclerosing cholan- tional Autoimmune Hepatitis Group Report:
gitis. J Hepatol. 2003;38:S24-S37. Review of criteria for diagnosis of autoim-
mune hepatitis. J Hepatol. 1999;31:929.
13. Valla D. The diagnosis and management of
the Budd-Chiari syndrome: consensus and 24. Perlmutter D. Liver injury in Alpha 1-antit-
controversies. Hepatology. 2003;38:793-803. rypsin deficiency. Clin Liver Dis. 2000;4:
387-408.
14. Tavill A. Diagnosis and managment of
hemochromatosis: AASLD Practice Guide- 25. Srinivasan P, Rela M, Prachalias A, et al.
lines. Hepatology. 2001;33:1321-1328. Liver transplantation for Budd-Chiari syn-
drome. Transplantation. 2002;73:973-977.
15. Kim WR, Lindor KD, Locke GR, et al. Epi-
demiology and natural history of primary bil- 26. Schilsky MK, Scheinberg JH, Sernleib I. Liver
iary cirrhosis in a US community. Gastroen- transplantation for Wilson’s disease: indica-
terology. 2000;119. tions and outcome. Hepatology. 1994;19:583-
587.
16. Sartin JS. Granulomatous hepatitis: A retro-
spective review of 88 cases at the Mayo 27. Tung BY, Farrell FJ, McCashland TM, et al.
Clinic. Mayo Clin Proc. 1991;66:914-918. Long-term follow-up after liver transplanta-
tion in patients with hepatic iron overload.
17. Valla DC. Hepatic granulomas and hepatic Liver Transpl Surg. 1999;5:369-374.
sarcoidosis. Clin Liver Dis. 2000;4:269-285.
28. Graziadei IW, Wiesner RH, Marotta PJ, et al.
18. Valla D, Degott C, et al. Hepatic sarcoidosis Long-term results of patients undergoing liver
with portal hypertension: a report of seven transplantation for primary sclerosing cholan-
cases with a review of the literature. Q J Med. gitis. Hepatology. 1999;30:1121-1127.
1987;63:531-544.
29. Stewart SF. The management of alcoholic
19. Harrison SA. Hereditary hemochromatosis: liver disease. J Hepatol. 2003;38:S2-S13.
Update for 2003. J Hepatol. 2003;38:S14-
S23.20. 30. Farrant JM, Hayllar KM, Wilkinson ML, et al.
Natural history and prognostic variables in pri-
20. Harnois DM, Angulo P, Jorgensen RA, mary sclerosing cholangitis. Gastroenterol-
Larusso NF, Lindor KD. High-dose ogy. 1991;100:1710-1717.
ursodeoxycholic acid as therapy for patients
with primary sclerosing cholangitis. Am J 31. Teli MR, James OF, Burt JA, et al. The natural
Gastroenterol. 2001;96:1558-1562. history of nonalcoholic fatty liver: A follow-
up study. Hepatology. 1995;22:1714-1719.
21. Shah AB, Chernov I, Zhang HT, et al. Identifi-
cation and analysis of mutations in the Wilson 32. Matteoni CA, Younossi JM, Gramlich T, et al.
disease gene (ATP7B): Population frequen- Nonalcoholic fatty liver disease: A spectrum
cies, genotype-phenotype correlation, and of clinical and pathological severity. Gas-
functional analyses. Am J Hum Genet. troenterology. 1999;116:1413-1419.
1997;61:317-328.
33. Sanyal AJ, Campbell-Sargent C, Mirshahi, et
22. Okuda K. Inferior vena cava thrombosis at its al. Nonalcoholic steatohepatitis: Association
hepatic portion (obliterative hepatocavopa- of insulin resistance and mitochondrial abnor-
thy). Semin Liver Dis. 2002;22:15-26. malities. Gastroenterology. 2001;120:1183-

282 EDUCATIONAL REVIEW MANUAL IN GASTROENTEROLOGY


1192. 43. Angulo P. Primary sclerosing cholangitis.
Hepatology. 1999;30:325-332.
34. Zeitoun G, Escolano S, Hadengue A, et al.
Outcome of Budd-Chiari syndrome: A multi- 44. Lindor KD. Primary sclerosing cholangitis.
variate analysis of factors related to survival In: Diseases of the Liver. Volume 1; 673-684.
including surgical portosystemic shunting.
Hepatology. 1999;30:84-89. 45. Ponsioen CY. Primary sclerosing cholangitis:
a clinical review. 1998;93:515-523.
35. Vergani D, Choudhuri K, Bogdanos DP,
Mieli-Vergani G. Pathogenesis of autoim- 46. Schilsky MK, Sernleib I. Prognosis of Wilso-
mune hepatitis. Clin Liver Dis. 2002 nianchronic active hepatitis. Gastroenterol-
Aug;6(3):727-737. ogy. 1991;100:762-767.

36. Britton RS, Fleming RE, Parkkila S, Waheed 47. Jones EA. The pruritus of cholestasis.
A, Sly WS, Bacon BR. Pathogenesis of hered- Hepatology. 1999;29:1003-1006.
itary hemochromatosis: genetics and beyond.
Semin Gastrointest Dis. 2002;13:68-79. 48. Sorbi D, Lindor KD. The ratio of aspartate
aminotransferase to alanine aminotransferase:
37. Kita H, Nalbandian G, Keeffe EB, Coppel RL, potential value in differentiating nonalcoholic
Gershwin ME. Pathogenesis of primary bil- steatohepatitis from alcoholic liver disease.
iary cirrhosis. Clin Liver Dis. 2003;7:821- Am J Gastroenterol. 1999;94:1018-1022.
839.
49. Wong PY, Portmann B, O’Grady JG, et al.
38. Akriviadis E, Botla R, Briggs W, Han S, Recurrence of primary biliary cirrhosis after
Reynolds T, Shakil O. Pentoxifylline liver transplantation following FK506-based
improves short-term survival in severe acute immunosuppression. J Hepatol. 1993;17:284-
alcoholic hepatitis: a double-blind placebo- 287.
controlled trial. Gastroenterology.
2000;119:1637-1648. 50. Ishak K. Sarcoidosis of the liver and bile
ducts. Mayo Clin Proc. 1998;73:467-472.
39. Olynyk JK, Cullen DJ, Aquilia S, Rossi E,
Summerville L, Powell LW. A population 51. Ueno T, Sugawara H, Sujaku K, et al. Thera-
based study of the clinical expression of the peutic effects of restricted diet and exercise in
hemochromatosis gene. N Engl J Med. obese patients with fatty liver. J Hepatol.
1999;341:718-724. 1997;27:
103-107.
40. Roberts EA. A practice guideline on Wilson
disease. Hepatology. 2003;37:1475-1492. 52. Locke GR 3rd, Therneau TM, Ludwig J,
Dickson ER, Lindor KD. Time course of histo-
41. Mitchell SA, Bansi DS, Hunt N, Von logical progression in primary biliary cirrho-
Bergmann K, Fleming KA, Chapman RW. A sis. Hepatology. 1996;23:52-56.
preliminary trial of high-dose ursodeoxycholic
acid in primary sclerosing cholangitis. Gas- 53. DeLeve LD, McDonald GB. Toxic injury to
troenterology. 2001;121:900-907. hepatic sinusoids: sinusoidal obstruction syn-
drome (veno-occlusive disease). Semin Liver
42. Olsson R, Danielsson A, Jarnerot G, et al. Dis. 2002;22:27-42.
Prevalence of primary sclerosing cholangitis
in patients with ulcerative colitis. Gastroen-
terology. 1991;100:1319-1323.

CHAPTER 8: NON-VIRAL LIVER DISEASE 283


54. Ganger DR, Klapman JB, McDonald V, et al.
Transjugular intrahepatic portosystemic shunt
(TIPS) for Budd-Chiari syndrome or portal
vein thrombosis: review of indications and
problems. Am J Gastroenterol. 1999;94:603-
608.

55. Olinski AT. Treating Budd-Chiari syndrome:


making rational choices from a myriad of
options. J Clin Gastroenterol. 2000;30:155-
161.

56. Hill GM, Brewer GJ, Prasad AS, Hydrick CR,


Hartmann DE. Treatment of Wilson’s disease
with zinc: I. Oral zinc therapy regimens.
Hepatology. 1987;7:522-528.

57. Beuers U, Spengler U, Ruis W, et al.


Ursodeoxycholic acid for treatment of pri-
mary sclerosing cholangitis: a placebo con-
trolled trial.
Hepatology. 1992;16:707-714.

58. Scheinberg IH, Sternleib I. The use of trien-


tine in preventing the effects of interrupting
penicillamine therapy in Wilson’s disease.
N Engl J Med. 1987;317:209-213.

59. Tanzi RE, Petrukhin K, Chernov I, et al. The


Wilson disease gene is a copper transporting
ATPase with homology to the Menkes disease
gene. Nat Genet. 1993:344-350.

60. Steindl P, Ferenci P, Dienes HP, et al. Wilson’s


disease in patients presenting with liver dis-
ease: a diagnostic challenge. Gastroenterol-
ogy. 1997;113:212-218.

61. Fairbanks KD, Tavill AS. Liver Disease In


Alpha-1 Antitrypsin Deficiency: A Review.
Am J Gastro. 2008;103:2136-2141.

62. Roberts EA, Schilsky ML. Diagnosis and


Treatment of Wilson’s Disease: An Update.
Hepatology. 2008;47:2089-2111.

284 EDUCATIONAL REVIEW MANUAL IN GASTROENTEROLOGY


12. Questions

1. A 34-year-old woman presents to her physician AST 75 WBC 18.1


with a one-month history of malaise and
anorexia. Her physical examination is remark- ALT 30 HCT 33.5
able only for mild nontender hepatomegaly.
Liver enzymes are as follows: ALP 141 MCV 103

AST 245 WBC 8.1 TB 15.5 PLT 98

ALT 280 HCT 33.5 ALB 3.7 PT 19 (control = 12)

ALP 110 MCV 87 GGT 976

TB 2.4 PLT 98 Liver ultrasound reveals normal bile ducts, no


masses, and splenomegaly with perisplenic
ALB 3.0 varices.

TP 9.7 Which of the following statements regarding his


condition is true?
Hepatitis B and C serologies are negative.
Iron studies are normal. Ultrasound shows a A. He meets criteria for corticosteroid therapy.
shrunken liver and mild splenomegaly, but no
masses or duct dilation. B. Severity of liver injury correlates directly with
height of ALT elevation.
Which of the following statements regarding this
condition is true? C. At this stage, abstinence from alcohol will not
affect outcome.
A. An antimitichondiral antibody test is usually
positive. D. Clinical disease is usually due to inherited
deficiency of alcohol dehydrogenase.
B. Interferon therapy is likely to induce remis-
sion. E. This condition affects 1%-3% of chronic
alcoholics.
C. Liver biopsy demonstrates PAS (+) granules
in hepatocytes.
3. A 41-year-old woman with a history of diabetes
D. Autoimmune thyroid disease is often a con- and hypercholesterolemia is referred to you for
comitant illness. evaluation of abnormal LFTs. Her exam is
remarkable for central obesity, and a liver edge
E. Phlebotomy will improve life expectancy. that is 2 cm below the right costal margin and
mildly tender to palpation. She has no other stig-
mata of chronic liver disease. Her liver enzymes
2. A 57-year-old male presents with 4 days of right are as follows:
upper quadrant pain, fever, and nausea. Two days
ago he developed frank jaundice. On exam, his AST 75 WBC 9.1
temperature is 100.5F, pulse 99, BP 135/70. He
has tender hepatomegaly 5 cm below the right ALT 80 HCT 39.5
costal margin and a palpable spleen tip. He has
frank jaundice and 1(+) pedal edema. He has a ALP 121 MCV 85
normal mental status and no asterixis. Labora-
tory results are as follows:

CHAPTER 8: NON-VIRAL LIVER DISEASE 285


TB 0.5 PLT 256 On further questioning, she reports a decreased
energy level over the past 3-4 months, and
ALB 4.0 PT 12 (control = 12) episodes of pruritus, especially at night.

GGT 100 Hb A1C 9.5% Which of the following statements regarding this
patient is correct?
TP 6.8
A. Liver ultrasound will show dilated intrahep-
An ultrasound reveals a mildly echogenic liver atic and extrahepatic bile ducts.
parenchyma and no other abnormalities.
B. Ursodiol therapy will likely be indicated for
Hepatitis serologies, iron studies, and autoim- this patient.
mune serologies are negative. Serum ceruloplas-
min and alpha-1 antitrypsin levels are normal. C. She should have a bone scan to determine the
She denies alcohol use, and has been on no new origin of her increased alkaline phosphatase.
medications in the past year.
D. She will likely have a high titer of anti-smooth
Which of the following statements is correct? muscle antibody.

A. She most likely has seronegative autoimmune E. Antimitochondrial antibody may be negative
hepatitis. in up to 50% of cases.

B. She will most likely have a positive anti-mito-


chondrial antibody. 5. A 36-year-old male with an eight-year history of
ulcerative colitis has the following liver
C. Better glycemic control may improve her liver chemistries:
chemistries.
AST 63 PLT 98
D. A Doppler ultrasound will reveal the etiology
of her abnormal liver enzymes. ALT 71 PT 12.3 (control = 12)

E. Chelation therapy is indicated for this patient. ALP 811 Serum AMA (-)

TB 0.5
4. A 48-year-old woman is seen for a routine physi-
cal and has liver enzymes drawn. The results are ALB 3.7
as follows:
GGT 1123
AST 53 WBC 9.1
An ultrasound is read as normal, with no masses
ALT 49 HCT 40.5 or ductal dilation.

ALP 645 MCV 86 Which of the following statements regarding this


patient is correct?
TB 0.8 PLT 189
A. Liver biopsy is usually diagnostic in this dis-
ALB 3.9 PT 12.1 (control = 12) order.

GGT 998 B. Serum p-ANCA is usually negative in this dis-


order.

286 EDUCATIONAL REVIEW MANUAL IN GASTROENTEROLOGY


C. An ERCP is indicated to make the diagnosis. E. If his 20-year-old son is found to have the
same condition, there is no benefit to treat-
D. The risk of cholangiocarcinoma in this disor- ment, as the progression of disease is unaf-
der is <1%. fected by therapy

E. This disorder usually has a benign clinical 7. A 48-year-old man presents with jaundice,
course. ascites, and pedal edema. Liver ultrasound
reveals a small nodular liver with perisplenic
6. A 58-year-old white male who has not seen a varices and splenomegaly. Liver enzymes are as
physician in “many years” is newly diagnosed follows:
with diabetes by his primary care physician. He
has also complained of shortness of breath on AST 31 PT 18 (control =12)
exertion, pedal edema, and decreased sexual
functioning. ALT 30 Hepatitis serologies negative

His laboratory studies are as follows: ALP 110 ANA (-)

AST 75 WBC 8.1 Serum Iron 234 TB 8.4 Iron studies normal

ALT 81 HCT 49.5 TIBC 288 ALB 2.9 Ceruloplasmin normal

ALP 101 MCV 88 Ferritin 2045 GGT 77 Alpha -1 AT level 30% of normal

TB 2.0 PLT 99 Which of the following statements regarding his


condition is correct?
ALB 2.9 PT 16 (control = 12)
A. Liver damage in this condition is caused by an
GGT 34 enzyme deficiency.

He has negative hepatitis serologies, takes no B. Alpha-1 antitrypsin replacement effectively


medications, and denies alcohol use. treats the liver and lung disease in patients
with this condition.
Ultrasound shows a shrunken, nodular liver and
splenomegaly. C. If the patient receives a liver transplant, the
disease will most likely recur in the trans-
Which of the following statements regarding his planted organ.
condition is correct?
D. The majority of patients with this condition
A. His clinical picture and laboratory studies are develop end stage liver disease.
most consistent with autoimmune hepatitis.
E. Liver biopsy will show characteristic granular
B. Genetic testing is unlikely to be revealing in deposits in hepatocytes.
this patient.

C. Orthotopic liver transplant for this disorder is 8. A 25-year-old woman with a history of
associated with >90% 5-year survival rates. schizophrenia is admitted to the hospital with
jaundice and confusion. Her liver chemistries are
D. There is a high rate of hepatocellular carci- as follows:
noma associated with this condition.
AST 45 WBC 10.1

CHAPTER 8: NON-VIRAL LIVER DISEASE 287


ALT 55 HCT 28.5 9. A 38-year-old woman is seen by her physician
for complaints of fatigue and mild shortness of
ALP 34 MCV 99 breath on exertion. She has a clear chest exam,
but on abdominal exam is found to have mild
TB 15.5 PLT 110 nontender hepatomegaly, but no other signs of
chronic liver disease. She is also noted to have
ALB 3.1 PT 19 (control = 12) cervical and axillary lymphadenopathy. She
takes no medications or supplements.
GGT 25 Haptoglobin undetectable
Liver enzymes are as follows:
Doppler ultrasound reveals an echogenic liver
parenchyma, no ductal abnormalities, and nor- AST 48 WBC 9.1
mal flow in the hepatic and portal veins.
ALT 51 HCT 37.5
Which of the following statements regarding the
most likely diagnosis is correct? ALP 422 MCV 88

A. Orthotopic liver transplant is the only effec- TB 1.1 PLT 199


tive treatment for this presentation.
ALB 4.4 PT 12.2 (control = 12)
B. The hemolytic anemia seen in this condition is
an autoimmune phenomenon. GGT 922

C. Acute chelation therapy is indicated and will Chest x-ray reveals bilateral hilar adenopathy.
improve outcome.
Liver ultrasound is read as normal.
D. A normal serum ceruloplasmin in this patient
rules out Wilson disease. Serum AMA is negative.

E. She likely has fulminant autoimmune Which of the following statements regarding the
hepatitis. likely diagnosis is true?

A. This condition is most likely due to drug


toxicity.

B. Her liver biopsy is likely to reveal caseating


granulomas.

C. Liver disease in this condition usually runs a


benign course.

D. Stool ova and parasite exam is likely to be


diagnostic.

E. The patient most likely has AMA-negative


PBC.

288 EDUCATIONAL REVIEW MANUAL IN GASTROENTEROLOGY


phosphatase and a normal ultrasound. Seronegative
autoimmune hepatitis is possible, but in patients with
Answers

1. D. a negative serologic workup for elevated transami-


The presence, in a young woman, of abnormal nase, greater than two-thirds have NAFLD, with only
transaminases, a normal alkaline phosphatase, and an a small fraction having occult autoimmune hepatitis.
elevated globulin fraction suggest autoimmune hep- In NAFLD, improved glycemic control may decrease
atitis. The negative viral serologies indicate that inter- transaminase levels.
feron will not be used to treat this patient. The normal
iron studies make hemochromatosis unlikely, so phle- 4. B.
botomy is not indicated. PAS(+) granules on liver The patient, a middle-aged female with fatigue, pruri-
biopsy are seen in alpha-1 antitrypsin deficiency, tus, and a markedly elevated alkaline phosphatase
which does not elevate serum globulin levels. Antim- with otherwise normal LFTs, likely has primary bil-
itochondrial antibodies are present in a small fraction iary cirrhosis (PBC). AMA testing is positive in >90%
of patients with autoimmune hepatitis, but are more of cases, while the anti-smooth muscle antibody may
characteristic of primary biliary cirrhosis. The best be weakly positive in a minority. Ultrasound testing is
answer is D., since a large number of patients with normal in patients with PBC, as the large bile ducts are
autoimmune hepatitis have a personal or family his- spared. A bone scan is unnecessary, as the elevated
tory of autoimmune disorders, with autoimmune thy- GGT confirms the liver origin of the elevated alkaline
roiditis the most commonly associated condition. phosphatase. Ursodiol is currently the recommended
therapy for PBC.
2. A.
The patient in question has severe alcoholic hepatitis, 5. C.
with the characteristic mild transaminitis with a 2:1 An elevated alkaline phosphatase in a patient with
ration of AST to ALT, marked elevation of bilirubin, inflammatory bowel disease suggests primary scleros-
and synthetic dysfunction with an elevated INR. The ing cholangitis. The negative ultrasound and negative
height of the transaminase elevation is NOT a marker AMA help exclude other etiologies of cholestatic liver
of disease severity in this condition. Abstinence from disease. A p-ANCA level is elevated in up to 80% of
alcohol is a key element in the treatment of alcoholic cases. The disease follows a progressive course
hepatitis, which occurs in 10%-35% of heavy alcohol toward end stage liver disease, and cholangiocarci-
users. The action of alcohol dehydrogenase is to con- noma occurs in 7%-15% of cases. Liver biopsy shows
vert ethanol to acetaldehyde, a toxic metabolite; thus, a characteristic lesion (“onion-skin fibrosis”) in only
ADH deficiency is not a factor in alcoholic hepatitis. 15% of cases; ERCP is the diagnostic test of choice,
The severity of alcoholic hepatitis is determined using demonstrating the characteristic beading and strictur-
the Discriminant Factor (DF), defined as [(4.6 x ing of the biliary tree.
PTPATIENT - PTCONTROL) + Total bilirubin].
When this factor is >32, steroid therapy should be 6. D.
considered. The DF in this patient is 47.7, meeting the The patient most likely has hereditary hemochro-
criteria for steroid use. matosis (HHC) with progression to cirrhosis. Autoim-
mune liver disease would not induce the abnormal
3. C. iron studies seen here. Genetic testing in Caucasian
This clinical scenario is most consistent with nonalco- patients with HHC is diagnostic in the majority of
holic fatty liver disease (NAFLD). Chelation therapy cases. Identification of affected family members is
is useful in Wilson disease, but the patient’s normal crucial, as phlebotomy clearly retards iron deposition
ceruloplasmin and clinical picture suggest against this and prevents the complications of HHC. The risk of
diagnosis. Doppler ultrasound may reveal portal hepatocellular carcinoma is significantly elevated in
hypertension, but it is diagnostic only in cases of patients with HHC and cirrhosis. Transplantation is
Budd-Chiari syndrome, which is unlikely given this associated with lower survival rates than for other
clinical picture. AMA is diagnostic of primary biliary conditions, thought to be due to the comorbid condi-
cirrhosis, which would present with a high alkaline tions usually present in patients with cirrhosis from
HHC.

CHAPTER 8: NON-VIRAL LIVER DISEASE 289


7. E.
The patient in question has alpha-1 antitrypsin defi-
ciency that has progressed to cirrhosis. Liver biopsy
will demonstrate PAS(+), diastase resistant granules
composed of abnormally processed protein. Liver dis-
ease occurs in 12%-15% of cases, and is due to an
excess of abnormally processed alpha-1 antitrypsin in
hepatocytes, not a deficiency of enzyme. Thus,
enzyme replacement, which can be effective in ame-
liorating the lung injury seen in this condition, does
not improve the liver dysfunction. Liver transplanta-
tion is curative, as the genetic defect in production of
alpha-1 antitrypsin is localized to the liver; the donor
liver is able to produce enzyme at normal levels.

8. A.
This case describes a young person who develops ful-
minant hepatic failure with mild elevations of
AST/ALT, marked hyperbilirubinemia, and poor syn-
thetic function. The past history of psychiatric illness,
and the combination of a very low alkaline phos-
phatase and hemolytic anemia, suggest Wilson dis-
ease (WD) as the etiology of her presentation. The
hemolysis in WD is due to the toxic effect of serum
copper on erythrocytes, rather than an autoimmune
process. Ceruloplasmin levels may be normal in this
presentation, since ceruloplasmin is an acute phase
reactant that is released in times of physiologic stress.
Chelation therapy, though very successful in the
chronic setting, is not effective in fulminant Wilsonian
hepatitis; liver transplantation in these cases is the
only effective therapy.

9. C.
This patient presents with cholestatic liver enzymes,
peripheral and hilar adenopathy, vague constitutional
symptoms, and dyspnea on exertion. The normal
ultrasound rules out metastatic malignancy and duct
obstruction. She has no exposure history to suggest
drug-induced cholestasis. Her AMA is negative, and
although AMA-negative PBC is possible, the hilar
and peripheral adenopathy make sarcoidosis a more
likely diagnosis. Ova and parasite examination would
not be useful, and a liver biopsy will show the charac-
teristic noncaseating granulomas. Liver disease in sar-
coidosis generally follows a benign course, although a
minority of cases may develop severe hepatic disease
and portal hypertension.

290 EDUCATIONAL REVIEW MANUAL IN GASTROENTEROLOGY

You might also like